File:  [Local Repository] / db / baza / ovsch14.txt
Revision 1.5: download - view: text, annotated - select for diffs - revision graph
Fri Jul 6 00:21:22 2018 UTC (5 years, 10 months ago) by rubashkin
Branches: MAIN
CVS tags: HEAD
https

Чемпионат:
Открытый Всероссийский синхронный чемпионат - 2014/15

Дата:
28-Sep-2014 - 01-Mar-2015

Тур:
Этап 1

Дата:
28-Sep-2014

Редактор:
Александр Коробейников (Саратов), при участии Владимира Цвингли (Москва)

Инфо:
Редактор благодарит за тестирование вопросов Дмитрия Великова, Ивана
Ефремова, Михаила Иванова, Тимура Кафиатуллина, Владислава Короля,
Екатерину и Николая Лёгеньких, Дениса Маркова, Максима Мерзлякова,
Наталию Новыш, Игоря Петрова и Михаила Савченкова.

Вопрос 1:
В Рио-де-Жанейро собираются построить так называемый Музей Завтра. Его
архитектор Сантьяго Калатрава замечает, что музей должен содержать
ПЕРВЫЕ, а не ВТОРЫЕ. Через минуту напишите ВТОРОЙ.

Ответ:
Ответ.

Комментарий:
Музей будущего должен не давать ответы, а ставить вопросы.

Источник:
   1. http://www.bbc.com/travel/blog/20111201-the-futurist-rios-museum-of-tomorrow
   2. Лекция Сантьяго Калатравы в институте "Стрелка", 22.06.2014 г.

Автор:
Владимир Цвингли (Москва)

Вопрос 2:
В турецкой пословице упоминается не вода, а йогурт. Какое действие с ним
совершают?

Ответ:
На него дуют.

Зачет:
По слову "дуть" или однокоренным к нему.

Комментарий:
Русская пословица - "Обжегшись на молоке, будешь дуть и на воду". В
турецкой пошли еще дальше: там упоминается продукт, всегда употребляемый
холодным.

Источник:
   1. http://turkishcrossroads.wordpress.com/2011/05/12/more-turkish-idioms/
   2. http://www.kolikler.com/liste/cevap/8281/yogurt/yogurtla-ilgili-deyim-ve-atasozleri-nelerdir
   3. http://dic.academic.ru/dic.nsf/michelson_new/6797/

Автор:
Владимир Цвингли (Москва)

Вопрос 3:
На одном логотипе буквы "l" [эль], "f" [эф] и "g" [джи] имеют вид
ПЕРВЫХ, а еще одна буква выполнена в виде ВТОРОЙ. Назовите ПЕРВУЮ и
ВТОРУЮ, каждую тремя словами.

Ответ:
Клюшка для гольфа, лунка для гольфа.

Комментарий:
Так слово "golf" изображено на эмблеме одного из гольф-клубов.

Источник:
http://www.adme.ru/tvorchestvo-dizajn/30-umnyh-logotipov-700760/

Автор:
Александр Коробейников (Саратов)

Вопрос 4:
В фильме Франсуа Озона похититель удерживает девушку в подвале. В одной
из сцен эта героиня долго смотрит на НЕЕ. ОНА - это секрет. Назовите ЕЕ.

Ответ:
Паутина.

Комментарий:
Положение девушки сходно с положением пойманного насекомого в паутине.
Секрет - это выделяемая животным жидкость, результат секреции.

Источник:
   1. Х/ф "Криминальные любовники" (1999), реж. Ф. Озон, 43:35.
   2. http://ru.wikipedia.org/wiki/Паутина

Автор:
Владимир Цвингли (Москва)

Вопрос 5:
Лирического героя одной из своих песен Ник Кейв называет Ларри и
утверждает, что этот герой смог добиться большего, чем Гарри Гудини. Под
каким именем этого Ларри знаем мы?

Ответ:
Лазарь.

Комментарий:
Лазарь смог воскреснуть и выбраться из могилы. Гарри Гудини известен
схожими трюками, но до уровня Лазаря ему далеко!

Источник:
http://www.bbc.co.uk/music/reviews/w8p2

Автор:
Владимир Цвингли (Москва)

Вопрос 6:
Петр I считал Карла XII надменным юнцом, не способным управлять страной.
Сюжет о каком герое стал главной декоративной темой оформления корабля
"Полтава"?

Ответ:
О Фаэтоне.

Комментарий:
Петр же себя ассоциировал с Зевсом, наказавшим самонадеянного мальчишку.
Фаэтон оказался не способен управлять колесницей своего отца Гелиоса и
потерпел крах, как и Карл XII под Полтавой.

Источник:
   1. http://ru.wikipedia.org/wiki/Полтава_(линейный_корабль,_1712)
   2. http://poltava1712.ru/sites/default/files/vse_risunki_dzheniya-14_0.jpg

Автор:
Александр Коробейников (Саратов)

Вопрос 7:
   <раздатка>
   It came with my Pea Sea.
   It plane lee marks four my revue
   Miss Steaks I can knot sea.
   </раздатка>
   Перед вами отрывок написанного в 1992 году стихотворения Джеррольда
ЗАра. Ответьте словом с двумя английскими корнями, недостатки чего хотел
продемонстрировать Зар.

Ответ:
Спеллчекера.

Зачет:
Spellchecker.

Комментарий:
Стихотворение составлено из набора слов, созвучного осмысленному тексту.
Первые спеллчекеры работали, сравнивая каждое из встреченных слов со
своим словарем, но не учитывая связь слов в предложении.

Источник:
http://en.wikipedia.org/wiki/Spell_checker

Автор:
Владимир Цвингли (Москва)

Вопрос 8:
Герой Андрея Гребенщикова говорит, что люди, родившиеся в метро, начисто
лишены перспективы, и упоминает ИКС. ИКСУ посвящена серия других
романов. Назовите ИКС двумя словами.

Ответ:
Плоский мир.

Комментарий:
В данном случае перспектива - это еще и умение видеть объем и
многогранность, а мир метро кажется герою двумерным, плоским. Плоский
Мир - место действия многих романов Терри Пратчетта.

Источник:
А. Гребенщиков. Обитель снов. http://www.flibusta.net/b/325458/read

Автор:
Артем Викторов (Саратов)

Вопрос 9:
В начале двадцатого века в Италии были популярны карандаши, красные с
одного конца и синие с другого. Что основал художник Джорджо МунджиАни,
придерживавшийся космополитических взглядов?

Ответ:
Клуб "Интернационале" (Милан).

Зачет:
По словам "Интернационале" или "Интер".

Комментарий:
Ориентируясь на форму уже существовавшего клуба "Милан", Мунджиани
оставил в ней черный цвет, но дополнил его синим в противовес красному
цвету "Милана".

Источник:
http://www.sports.ru/tribuna/blogs/marinamargarita/669690.html

Автор:
Александр Коробейников (Саратов)

Вопрос 10:
Герой одного сериала - шахматист по фамилии БАлаган, который играет
только по Интернету. Его соперники, не знавшие его в лицо, дали ему
прозвище, заменив в его фамилии один слог двумя другими. Напишите это
прозвище.

Ответ:
Балаклава.

Комментарий:
Маски-балаклавы закрывают лицо, оставляя только глаза.

Источник:
Телесериал "Шах и мат", 6-я серия.

Автор:
Александр Коробейников (Саратов)

Вопрос 11:
Известный путешественник Грэм Хьюз уверяет, что без этой тяжелой, а
порой мокрой и дурно пахнущей штуки можно и обойтись. Что же Хьюз,
вопреки известному источнику, рекомендует не брать с собой?

Ответ:
Полотенце.

Комментарий:
В отличие от Дугласа Адамса, автора "Автостопом по Галактике", который
писал, что полотенце - неотъемлемый элемент экипировки автостопщика,
Хьюз считает, что без него легко обойтись.

Источник:
"Discovery", 2014, N 5. - С. 103.

Автор:
Александр Коробейников (Саратов)

Вопрос 12:
Дизайнер ДжОшуа РенУф совместил будильник с НЕЙ, и просыпается хозяин
этого агрегата не только от звукового сигнала. Назовите ЕЕ двукоренным
словом.

Ответ:
Кофемашина.

Зачет:
Кофеварка; кофемолка.

Комментарий:
К пробуждению автомат готовит чашечку ароматного кофе, от запаха
которого хозяин и просыпается. Впрочем, звуковой сигнал для контроля
тоже есть, хотя и довольно тихий. Возможно, в перерыве некоторые игроки
успеют попить кофе.

Источник:
http://www.adme.ru/tvorchestvo-dizajn/etot-budilnik-nalet-vam-s-utra-chashechku-kofe-736610/

Автор:
Александр Коробейников (Саратов)

Вопрос 13:
В пояснительной табличке к перламутровой чаше на одной выставке автор
вопроса увидел название, получившее широкую известность в СССР в
середине 80-х годов. Напишите это название.

Ответ:
Nautilus [Pompilius].

Зачет:
"Наутилус [Помпилиус]".

Комментарий:
Табличка сообщала, что для перламутровых инкрустированных чаш обычно
использовали раковину моллюска вида Nautilus pompilius. Советская и
российская рок-группа в 1985 году сменила название с "Наутилус" на
"Nautilus Pompilius".

Источник:
   1. Выставка "Сокровищница Медичи. Из собрания Музея Серебра и других
музеев Флоренции".
   2. http://www.kreml.ru/exhibitions/moscow-kremlin-exhibitions/sokrovishchnitsa-medichi-iz-sobraniya-muzeya-serebra-i-drugikh-muzeev-florentsii/
   3. А.К. Троицкий. Рок-музыка в СССР. - М.: Книга, 1990. - С. 239-240.

Автор:
Владимир Цвингли (Москва)

Вопрос 14:
Герой документального фильма в юности был членом Армии Крайовой. По
словам Михаила ТрофимЕнкова, он выжил, а не обратился в ПЕРВЫЙ, чтобы
возродиться ВТОРЫМ. Назовите ПЕРВЫЙ и ВТОРОЙ.

Ответ:
Пепел, алмаз.

Комментарий:
Герой фильма "Пепел и алмаз", как и герой описанного в вопросе
документального фильма "Тихая заводь", тоже входил в антифашистское и
антикоммунистическое польское подполье, но погиб.

Источник:
http://seance.ru/n/21-22/vertigo-2/prezumptsiya-vinovnosti/

Автор:
Владимир Цвингли (Москва)

Вопрос 15:
Игорь Можейко пишет, что своей славой ОН обязан тому, что сделал всё сам
- без помощи армии, жрецов и аппарата принуждения. Назовите ЕГО имя.

Ответ:
Герострат.

Комментарий:
Храмы сжигали и раньше, но Герострат обошелся собственными силами.

Источник:
И.В. Можейко. 7 и 37 чудес. http://www.flibusta.net/b/351970/read

Автор:
Александр Коробейников (Саратов)

Вопрос 16:
Герой одного романа, чтобы устрашить преследующих его ночных животных,
делает с одним из них то же, что известный человек, по его собственным
словам, сделал по ошибке. Назовите этого человека.

Ответ:
[Оззи] Осборн.

Комментарий:
Эти существа - летучие мыши, одной из которых герой и откусывает голову.
Подобным же поступком известен и Оззи Осборн, хотя, по его собственному
мнению, считал, что летучая мышь резиновая.

Источник:
Р. Глушков. Грань Бездны. http://www.litmir.net/br/?b=154646&p=71

Автор:
Артем Викторов (Саратов)

Вопрос 17:
В Нью-Йорке находятся штаб-квартиры и представительства многих
американских компаний, работающих в сфере компьютерных технологий. Хотя
они расположены и не на одной улице, этот конгломерат получил состоящее
из двух слов прозвище. Напишите его по-английски.

Ответ:
Silicon Alley [сИликон Элли].

Комментарий:
Буквально - "Кремниевая аллея", по аналогии с Кремниевой долиной
(Silicon Valley) в Калифорнии.

Источник:
http://en.wikipedia.org/wiki/Silicon_Alley

Автор:
Владимир Цвингли (Москва)

Вопрос 18:
Во время оккупации Варшавы немцами герою книги Джека Майера удалось
раздобыть кусок дефицитного угля, который его подруга сравнила с НЕЙ.
Другую "ЕЕ" сыграл "Сансет". Назовите ЕЕ двумя словами.

Ответ:
Черная жемчужина.

Комментарий:
Уголь представлял собой настоящую драгоценность. Корабль "Черная
жемчужина" из франшизы "Пираты Карибского моря" создан на основе корабля
HMS Sunset.

Источник:
   1. Дж. Майер. Храброе сердце Ирены Сендлер.
http://www.flibusta.net/b/351708/read
   2. http://pirates.wikia.com/wiki/Sunset_(ship)

Автор:
Александр Коробейников (Саратов)

Вопрос 19:
Во время прОводов родственника герой одного романа называет ИКСА злобным
таксистом. Какое имя мы заменили ИКСОМ?

Ответ:
Харон.

Комментарий:
Родственника провожали в последний путь и основательно потратились.

Источник:
А. Гребенщиков. Обитель снов. http://www.flibusta.net/b/325458/read

Автор:
Артем Викторов (Саратов)

Вопрос 20:
Алексей Леонов был зачислен в первый отряд космонавтов и, конечно,
мечтал первым отправиться в космос. Интересно, что среди его картин есть
историческое полотно "ИКС в Антарктиде". Назовите ИКС.

Ответ:
Шлюп "Восток".

Зачет:
Корабль "Восток"; "Восток".

Комментарий:
Леонов наверняка хотел полететь на космическом корабле "Восток" (а в
итоге слетал на "Восходе-2"). Шлюпом "Восток" командовал Беллинсгаузен,
открыватель Антарктиды.

Источник:
   1. http://www.seaexpo.ru/ru/miscellaneous/company-news/1_849.html
   2. http://pyatkovvova.narod.ru/index/0-381

Автор:
Александр Коробейников (Саратов)

Вопрос 21:
Рэпер Мистер Си в одной песне собирается на завтрак залить себе овсяные
хлопья ИМ. Согласно одному мифу, ОН появился из-за того, что героине
было тяжело каждый день возвращаться домой пешком. Назовите ЕГО.

Ответ:
Млечный Путь.

Комментарий:
Рэпер несколько преувеличивает: он хочет просто много молока. Героиня
мифа создала реку и с тех пор плавала по ней в лодке.

Источник:
   1. http://webkind.ru/text/967964471_1556824p524440981_text_pesni_pentimento-bozze-music.html
   2. http://www.myfhology.info/stella-myth/milk-way.html

Автор:
Артем Викторов (Саратов)

Вопрос 22:
[Ведущему: не сообщать о наличии отточия, оно не имеет значения для
вопроса.]
   Выражение "Во всём виноваты евреи и велосипедисты" происходит из
романа Ремарка "Черный обелиск". Вот цитата из романа:
   - При чем тут велосипедисты? - <...> удивляется Генрих.
   - А при чем тут евреи? - ДЕЛАЕТ ЭТО Ризенфельд.
   Какие четыре слова мы заменили словами "ДЕЛАЕТ ЭТО"?

Ответ:
Отвечает вопросом на вопрос.

Комментарий:
Забавно, что Ризенфельд, защищающий евреев в диалоге, делает то же, что
в обыденном представлении часто делают евреи.

Источник:
Э.М. Ремарк. Черный обелиск. http://www.flibusta.net/b/326124/read

Автор:
Владимир Цвингли (Москва)

Вопрос 23:
Родовитый князь из романа Евгения Водолазкина не имел детей, так что
образно называл себя ИКСОМ. ИКС упоминается в Евангелиях от Луки и
Матфея. Назовите ИКС одним словом.

Ответ:
Сучок.

Комментарий:
Князь называл себя сучком на генеалогическом древе своего рода.
Противопоставление сучка в чужом глазу и бревна в собственном восходит к
Евангелию от Луки.

Источник:
   1. Е.Г. Водолазкин. Похищение Европы.
http://www.flibusta.net/b/338557/read
   2. http://www.allbible.info/bible/sinodal/lu/6/
   3. http://www.allbible.info/bible/sinodal/mt/7

Автор:
Александр Коробейников (Саратов)

Вопрос 24:
Персонаж одного сериала отмечает, что у НИХ прибавилось работы после
Первой мировой войны. ФрЕдерик МАйерс считал, что ИХ работа связана с
автоматическим проявлением подсознательного. Назовите ИХ.

Ответ:
Медиумы.

Зачет:
Спириты.

Комментарий:
Первая мировая война принесла множество смертей, и многие захотели
пообщаться со своими погибшими родными и близкими. Майерс считал, что
многие явления деятельности медиума связаны с автоматическим проявлением
его подсознания.

Источник:
   1. Телесериал "Леди-детектив Фрайни Фишер", s02e01.
   2. http://ru.wikipedia.org/wiki/Медиум

Автор:
Александр Коробейников (Саратов)

Вопрос 25:
Полина Павлова пишет, что на выборах некоторые кандидаты, надеясь на
одобрение избирателей, подчеркивают, что у них имеется ЭТО. ЭТО имеет
"команда". Назовите ЭТО.

Ответ:
[Буквосочетание] ДА.

Комментарий:
КандиДАт предлагает сказать себе "ДА", при этом особенно везет
кандидатам, которые имеют нужное сочетание букв и в фамилии. Часто
спекулируют и на добавлении в лозунги слова "правДА". В слове "команда"
тоже есть слог "ДА".

Источник:
http://www.tverlife.ru/news/11181.html

Автор:
Илья Токарев (Заречный)

Вопрос 26:
Кличка призрака, якобы живущего в известном здании, - сокращение от слов
"демонический кот". В каком городе находится это здание?

Ответ:
Вашингтон.

Комментарий:
Кличка - DC [ди-си], как и сокращенное название округа Колумбия, в
котором находится Вашингтон. Призрак кота якобы периодически появляется
в Белом доме.

Источник:
http://sfilm.org/articles/010/демонический-кот-и-другие-привидения/

Автор:
Александр Коробейников (Саратов)

Вопрос 27:
Во время Первой мировой войны ЗИгберт ТАрраш написал пародию на
шовинистическую истерию на примере анализа шахматных дебютов. Так, он
подверг критике французскую защиту и английское начало. Еще один дебют
Тарраш охарактеризовал восторженно и дал ему название, которое всего на
одну букву длиннее реально существующего названия другого дебюта.
Напишите это новое название.

Ответ:
Прусская партия.

Комментарий:
"Прусской партией" Тарраш в духе немецкого патриотизма назвал защиту
двух коней (впрочем, мог бы назвать так и любой другой дебют). Русскую
партию он в статье, естественно, тоже критикует.

Источник:
Ю.Л. Авербах. О чем молчат фигуры. http://www.flibusta.net/b/236155/read

Автор:
Владимир Цвингли (Москва)

Вопрос 28:
Герои Йосипа НовАковича выдернули почти все флажки с уличных растяжек
перед первомайской демонстрацией. Растяжки после этого сравниваются с
АЛЬФАМИ старика. Действие "АЛЬФ" происходит в основном на острове.
Назовите АЛЬФЫ.

Ответ:
Челюсти.

Комментарий:
Остались только некоторые флажки, как редкие зубы во рту у старика.
Неудивительно, что в фильме про акул место действия - остров.

Источник:
   1. Й. Новакович. День дурака. http://www.flibusta.net/b/273308/read
   2. http://www.kinopoisk.ru/film/396/

Автор:
Александр Коробейников (Саратов)

Вопрос 29:
Герой одного фильма сначала выиграл много денег у бандитов, а затем
снова встретился с ними за карточным столом. Герой с горькой иронией и
вместе с тем с оптимизмом заметил, что он - ИКС. Какое слово, оба корня
в котором латинские, мы заменили ИКСОМ?

Ответ:
Амбидекстр.

Комментарий:
Бандиты из мести сломали герою руку, однако он, по счастью, владеет
другой рукой так же хорошо. Слово "амбидекстр" происходит от слов "оба"
и "правый". Хотя автомат "однорукий бандит" никакого отношения к вопросу
не имеет, но может помочь его взять.

Источник:
Х/ф "Город грехов - 2" (2014), реж. Ф. Миллер, Р. Родригес.

Автор:
Артем Викторов (Саратов)

Вопрос 30:
В статье пятой Уголовно-процессуального кодекса России даны определения
слов и выражений, использующихся в кодексе. Назовите несклоняемым словом
определяемое в этой статье понятие, которое ни в одном другом месте
кодекса не встречается.

Ответ:
Алиби.

Комментарий:
Раньше оно встречалось еще и в части 6 статьи 234 кодекса. Однако
законом от 3 июня 2006 года статья была признана утратившей силу, а
определение осталось. "Alibi" переводится с латыни как "в другом месте".

Источник:
   1. http://www.zakonrf.info/upk/5/
   2. Поиск по: Уголовно-процессуальный кодекс Российской Федерации. СПС
"Консультант Плюс".

Автор:
Владимир Цвингли (Москва)

Вопрос 31:
В 2013 году Национальный исторический музей Румынии провел серию
мероприятий под названием "ОНА MCM [эм-це-эм]". Назовите ЕЕ.

Ответ:
Колонна.

Зачет:
Колонна Траяна, Траянова колонна.

Комментарий:
Праздновалась 1900-летняя годовщина со времени сооружения колонны
Траяна.

Источник:
http://www.mnir.ro/index.php/columna-mcm-celebrarea-a-1900-de-ani-de-la-ridicarea-columnei-lui-traian/

Автор:
Александр Коробейников (Саратов)

Вопрос 32:
На концепцию ресторана "Twin Stars" [твин старз] его создателя Алексея
Ходорковского натолкнуло королевство. Какое именно?

Ответ:
Кривых зеркал.

Комментарий:
В ресторане все официантки и бармены - близнецы.

Источник:
http://www.uznayvse.ru/ekonomika/v-stolichnom-restorane-twin-stars-posetiteley-obsluzhivayut-bliznetsyi-55475.html

Автор:
Александр Коробейников (Саратов)

Вопрос 33:
Леонид Костюков называет бессмысленные, повторяющие одно и то же
рассуждения доступными. Какие две буквы мы пропустили в этом вопросе?

Ответ:
во.

Комментарий:
Рассуждающий словно воду в ступе толчет.

Источник:
http://ps.1september.ru/article.php?ID=200005708

Автор:
Александр Коробейников (Саратов)

Вопрос 34:
Собеседник героини Нила Стивенсона хочет говорить только о технике, а
когда речь заходит, например, о стихах, умолкает. Какой фамилией этот
собеседник представляется?

Ответ:
Тьюринг.

Комментарий:
На самом деле собеседник - компьютер.

Источник:
http://www.erlib.com/Нил_Стивенсон/Алмазный_век,_или_Букварь_для_благородных_девиц/26/

Автор:
Александр Коробейников (Саратов)

Вопрос 35:
Герой Сири Джеймс предлагает героине начать всё сначала - так сказать,
СДЕЛАТЬ ЭТО. ЭТО часто ДЕЛАЮТ, например, перед русским языком. Что такое
"СДЕЛАТЬ ЭТО"?

Ответ:
Вытереть доску.

Зачет:
Стереть с доски.

Комментарий:
Так сказать, изготовить tabula rasa.

Источник:
С. Джеймс. Тайные дневники Шарлотты Бронте.
http://www.flibusta.net/b/298085/read

Автор:
Александр Коробейников (Саратов)

Вопрос 36:
Латинское выражение "сказано с НЕГО" обозначает какое-то темное и
непонятное выражение. Мистики утверждают, что ОН символизирует
восходящее, полуденное и заходящее солнце. Назовите ЕГО.

Ответ:
Треножник.

Комментарий:
Дельфийская пифия вещала с треножника и давала как можно более
обтекаемые и туманные ответы. На них мы, кстати, и намекаем вторым
фактом: что имеют в виду астрологи, не очень понятно - на то они и
астрологи, зато сущностей здесь три.

Источник:
   1. http://dic.academic.ru/dic.nsf/michelson_old/15570/
   2. http://www.vrata11.ru/index.php?option=com_content&task=view&id=232&Itemid=74

Автор:
Александр Коробейников (Саратов)

Тур:
Этап 2

Дата:
26-Oct-2014

Редактор:
Александр Кудрявцев (Николаев)

Инфо:
Автор игрового пакета благодарит за тестирование и ценные замечания:
Дмитрия Алёхина, Тимура Барского, Евгения Быстрова, Александра Ваксмана,
Галину Воловник, Владислава Говердовского, Владимира Городецкого,
Евгения Громова, Людмилу Губаеву, Антона Гусакова, Андрея Данченко,
Глеба Дрепина, Екатерину Дубровскую, Игоря Мазина, Вадима Молдавского,
Дмитрия Овчарука, Наталью Руберте, Дмитрия Слоуща, Екатерину Сосенко и
Дмитрия Чеснокова. Персональная благодарность Александру Коробейникову и
Максиму Мерзлякову.

Вопрос 1:
По мнению героя романа "2001: Космическая одиссея", эти пять цифр
слишком дорого стоили человеческим нервам. Напишите эти цифры в
правильном порядке.

Ответ:
5, 4, 3, 2, 1.

Зачет:
4, 3, 2, 1, 0.

Комментарий:
Герой Кларка имеет в виду обратный отсчет при запуске космического
корабля.

Источник:
Артур Кларк. 2001: Космическая одиссея.

Автор:
Александр Кудрявцев (Николаев)

Вопрос 2:
Эти аппараты состоят из нагревателя, в котором плавится сырье,
крутящейся головки, выбрасывающей волокно, и емкости-улавливателя. Что
производят с помощью этих аппаратов?

Ответ:
Сахарную вату.

Зачет:
Сладкую вату.

Источник:
"Галилео", 04.05.2007 г.

Автор:
Александр Кудрявцев (Николаев)

Вопрос 3:
Среди причин исчезновения голубых ара - резкое увеличение численности
диких ИХ, заселивших привычные для этого вида попугаев места
гнездования. ОНИ дали название модели аппарата для производства сахарной
ваты. Назовите ИХ.

Ответ:
Пчелы.

Комментарий:
Завезенные в Америку африканские пчелы захватили все дупла, в которых
привыкли устраивать гнезда голубые ара. Аппарат получил название
"Пчелка".

Источник:
   1. http://ru.wikipedia.org/wiki/Голубой_ара
   2. http://www.food-service.com.ua/apparat-saharnoy-vaty-pchelka-e-p71326

Автор:
Александр Кудрявцев (Николаев)

Вопрос 4:
Названием детища известной IT-компании [ай-ти-компании] стал глагол,
восходящий к слову со значением "свеча". Это не преминули подметить
язвительные комментаторы, упомянув известную антиутопию. Назовите
упомянутое детище.

Ответ:
(Amazon) Kindle [амазон киндл].

Комментарий:
"Amazon Kindle" - электронная книга компании "Amazon". Глагол "kindle",
означающий "зажигать огонь", происходит от древнескандинавского
"kyndill" [киндил], которое, кстати, не имеет отношения к латинскому
"candela" [кандела]. Создатели имели в виду свет знаний, который несут
книги. Комментаторы шутили, что бумажные книги теперь можно сжечь, и
упоминали антиутопию "451 градус по Фаренгейту".

Источник:
   1. http://www.etymonline.com/index.php?term=kindle&allowed_in_frame=0
   2. http://nancyfriedman.typepad.com/away_with_words/2008/12/how-the-kindle-got-its-name.html

Автор:
Александр Кудрявцев (Николаев)

Вопрос 5:
Считается, что прообразом ИКСА стала МавронЕри, населенная
микроорганизмами, выделяющими смертельно опасные токсины. Назовите ИКС
односложным словом.

Ответ:
Стикс.

Комментарий:
МавронЕри - река на Пелопоннесе, название которой переводится как
"Черная вода".

Источник:
   1. http://en.wikipedia.org/wiki/Mavroneri
   2. http://en.wikipedia.org/wiki/Calicheamicin
   3. http://listverse.com/2014/09/29/10-real-world-entrances-to-mythical-locations/

Автор:
Александр Кудрявцев (Николаев)

Вопрос 6:
В телепрограмме "Человеческий фактор" ЭТО было названо надводной частью
айсберга. "Подводную" часть составляют, в том числе, три однотипных
элемента. Назовите тот, название которого не содержит уменьшительного
суффикса.

Ответ:
Наковальня.

Комментарий:
ЭТО - ушная раковина. Молоточек, наковальня и стремечко - косточки
слухового аппарата.

Источник:
Человеческий фактор. Звуки музыки.

Автор:
Александр Кудрявцев (Николаев)

Вопрос 7:
[Ведущему: четко прочитать окончание в слове "ВТОРОЕ".]
   На исторической картине Клавдия Лебедева перед алчным князем стоит
склонившийся ПЕРВЫЙ, а на рисунке, изображающем последовавшие события, -
склоненные ВТОРЫЕ. Слова "ПЕРВЫЙ" и "ВТОРОЕ" происходят от общего корня.
Какие слова мы заменили словами "ПЕРВЫЙ" и "ВТОРОЕ"?

Ответ:
Древлянин, дерево.

Комментарий:
На картине Лебедева запечатлен князь Игорь, собирающий полюдье с
древлян. На рисунке изображена казнь Игоря, которого разорвали, привязав
к склоненным деревьям. Древлянам, по объяснению летописца, их имя дано
потому, что они жили в лесах.

Источник:
   1. http://ru.wikipedia.org/wiki/Игорь_Рюрикович
   2. http://kzeteron.ucoz.ru/photo?photo=21
   3. http://ru.wikipedia.org/wiki/Древляне

Автор:
Александр Кудрявцев (Николаев)

Вопрос 8:
Замкнутые дети - не единственная проблема детских психологов. К примеру,
когда поведение шестилетней Салиши Джонсон заставило администрацию
детского сада беспомощно развести руками, были использованы ОНИ.
Текстильные ОНИ имеют прочность на разрыв не менее ста сорока
килограммов. Назовите ИХ.

Ответ:
Наручники.

Комментарий:
Салиша Джонсон безобразничала в детском саду, и администрация вызвала
полицию. Стражам порядка пришлось замкнуть наручники на запястьях
маленькой хулиганки.

Источник:
   1. http://lenta.ru/news/2012/04/17/kindergartner/
   2. http://www.euro-security.info/euro-security.info/ru/textile-disposable-handcuffs.html

Автор:
Александр Кудрявцев (Николаев)

Вопрос 9:
[Ведущему: не сообщать игрокам, что слово "мостов" написано в кавычках.]
   Необычайное совпадение заставило ЕГО предложить вместо "мостов"
подвижную модель. Назовите ЕГО.

Ответ:
[Альфред ЛОтар] ВЕгенер.

Комментарий:
Ранее считалось, что общность флоры и фауны удаленных один от другого
материков объясняется существованием в далеком прошлом "сухопутных
мостов", связывавших материки. Вегенер обратил внимание на необычайное
совпадение их береговых линий и предложил теорию дрейфа материков.

Источник:
http://ru.wikipedia.org/wiki/Теория_дрейфа_материков

Автор:
Александр Кудрявцев (Николаев)

Вопрос 10:
Внимание, в тексте вопроса мы немного изменили одно из слов.
   На обложке альбома метал-группы "Австралийская машина убийств"
известный персонаж восседает на горе из черепов. Назовите этого
персонажа.

Ответ:
Терминатор.

Зачет:
T-800.

Комментарий:
Группа называется "Австрийская машина убийств", на обложках ее альбомов
изображен терминатор, в чертах которого легко угадывается Арнольд
Шварценеггер.

Источник:
   1. http://www.metalinjection.net/upcoming-releases/tim-lambesis-austrian-death-machine-triple-brutal-gets-a-release-date-new-song-released
   2. http://ru.wikipedia.org/wiki/T-800_(робот)

Автор:
Александр Кудрявцев (Николаев)

Вопрос 11:
(pic: 20140514.jpg)
   На снимке - одно из НИХ, так что даже самым большим скептикам
придется признать, что ОНИ - не вымысел. Назовите ИХ двумя словами.

Ответ:
Кладбища слонов.

Зачет:
Слоновьи кладбища.

Комментарий:
Снимок сделан в Таиланде. Существует миф, что слоны хоронят своих
умерших сородичей в особых местах, называемых слоновьими кладбищами.

Источник:
   1. http://www.avivas.ru/photo_view/77884.html
   2. http://www.pravda.ru/science/useful/11-01-2013/1140703-elephants_necro-0/

Автор:
Александр Кудрявцев (Николаев)

Вопрос 12:
Недавно служители одного зоопарка услышали доносившиеся из львиного
вольера крики о спасении. Однако кричавший человек показался львам
неубедительным и потерпел предсказуемую неудачу, так и не сумев... Что
именно сделать?

Ответ:
Обратить львов в христианство.

Зачет:
Крестить львов; сделать львов христианами.

Комментарий:
Некий Чен Чун Хо из Тайбея, возжелав, чтобы львы стали добрыми
христианами, прыгнул в львиный вольер и закричал: "Иисус спасет вас!".

Источник:
http://www.newsru.com/world/04nov2004/lion.html

Автор:
Александр Кудрявцев (Николаев)

Вопрос 13:
Гюнтер Лав как-то решил продать инструмент, дважды приносивший ему
первое место на международных конкурсах. В объявлении Гюнтер указал, что
тот в отличном состоянии, не имеет заметных повреждений и стоит всего
полторы тысячи евро. Назовите заглавного героя, имеющего
основополагающее сходство с этим инструментом.

Ответ:
Человек-невидимка.

Комментарий:
Гюнтер Лав продавал невидимую гитару. Конкурсы по игре на невидимой
гитаре проводятся уже около тридцати лет.

Источник:
http://strannovosti.ru/strange/humans/2199

Автор:
Александр Кудрявцев (Николаев)

Вопрос 14:
Известный итальянец в 1877 году сделал рисунок, весьма далекий от
оригинала, а интерпретируя нарисованное, использовал термин, вызвавший
всеобщее оживление. Каким бранным словом назвал за это упомянутого
итальянца автор вопроса?

Ответ:
Каналья.

Комментарий:
Известный итальянец - астроном Джованни Скиапарелли, "первооткрыватель"
марсианских каналов. Скиапарелли линии на своем рисунке назвал
итальянским словом "canali" [канАли], которое обозначает протоки как
искусственного, так и естественного происхождения. При переводе на
английский было использовано слово "canals" [кЭнэлс], обозначающее
исключительно каналы искусственного происхождения.

Источник:
"Вселенная" (The Universe), s01e02. "Марс: Красная планета".

Автор:
Александр Кудрявцев (Николаев)

Вопрос 15:
Внимание, в вопросе имеется некоторая натяжка.
   Прием, применявшийся при постановке поединков в фильмах "Матрица" и
"Убить Билла", позволил выделить эти фильмы в поджанр, который получил
название "ТАКОЕ кунг-фу". Назовите праздничный атрибут, снабжаемый ТАКОЙ
уздечкой.

Ответ:
Бутылка шампанского.

Зачет:
Шампанское; пробка [от] шампанского.

Комментарий:
Этот поджанр - wire-fu [вАйер фу], название которого можно перевести и
как "кунг-фу на тросах", и как "проволочное кунг-фу". С помощью тросов в
названных фильмах снимались головокружительные схватки. Проволочная
уздечка-мюзле удерживает пробку шампанского.

Источник:
   1. http://en.wikipedia.org/wiki/Wire_fu
   2. http://ru.wikipedia.org/wiki/Мюзле

Автор:
Александр Кудрявцев (Николаев)

Вопрос 16:
На сайте forumklassika.ru [форум классика точка ру] в шутку
утверждается, что появление опорного шпиля способствовало эмансипации
женщин, ведь раньше пышные одеяния мешали женщинам заниматься ЭТИМ.
Назовите ЭТО тремя словами.

Ответ:
Игра на виолончели.

Комментарий:
Раньше виолончель удерживали между ног, что не совсем удобно в пышных
юбках.

Источник:
http://www.forumklassika.ru/showthread.php?t=92900

Автор:
Александр Кудрявцев (Николаев)

Вопрос 17:
[Ведущему: о кавычках не сообщать.]
   ЕЕ начинают "доить" в семь лет, а по достижении двадцати пяти лет,
когда качество "молока" ухудшается, поступают самым беспощадным образом.
Назовите ЕЕ одним словом.

Ответ:
Гевея.

Комментарий:
Когда возраст дерева-каучуконоса достигает 25 лет и качество млечного
сока становится неудовлетворительным, гевею обычно срубают.

Источник:
Программа "В поисках приключений", посвященная Индии.

Автор:
Александр Кудрявцев (Николаев)

Вопрос 18:
Этот эффектный и рискованный прием, часто демонстрируемый на
специализированных шоу, у нас называют "свеча", а в англоязычных странах
- "wheelie" [вИли]. Что укорачивают, чтобы во время выполнения этого
приема избежать контакта с покрытием?

Ответ:
Выхлопную трубу.

Комментарий:
Свеча - это постановка мотоцикла на заднее колесо в положение "12
часов". Чтобы выхлопные трубы не царапали дорожное покрытие, их
необходимо подпилить. Термин "wheelie" - уменьшительное от "wheel"
[вил], что значит "колесо".

Источник:
http://www.r6-club.ru/forums/index.php?showtopic=547&st=20

Автор:
Александр Кудрявцев (Николаев)

Вопрос 19:
[Ведущему: о кавычках не сообщать.]
   Прекраснейший бриллиант КондЕ обнаружила голодная горничная,
прибиравшаяся в номере, в котором жили его похитители. Украденный камень
отыскался в ИКСЕ. В "ИКСЕ" можно отыскать Александра Александровича.
Назовите ИКС.

Ответ:
Яблоко.

Комментарий:
Преступники засунули бриллиант в яблоко, которым решила полакомиться
горничная. В слове "яблоко" есть буквосочетание "блок". Слово
"прекраснейший" - подсказка.

Источник:
   1. http://www.masteruvelir.ru/spravochnik-yuvelira/samyie-znamenityie-kamni/conde.html
   2. Тщательный анализ слова "яблоко".

Автор:
Александр Кудрявцев (Николаев)

Вопрос 20:
Безработный из Бронкса Майкл Перес разрисовал себя черным маркером за
несколько минут до выхода на сцену, где должна была состояться
торжественная церемония. Что купил Перес двумя неделями ранее?

Ответ:
Лотерейный билет.

Комментарий:
Майкл Перес, прежде чем выйти на сцену, где ему должны были вручить чек
на 14 миллионов долларов, нарисовал маркером бородку и усы. Победитель,
живший в неблагополучном районе, не хотел, чтобы его узнали проживавшие
по соседству бандиты.

Источник:
http://www.nypost.com/2008/04/16/hidden-wealth/

Автор:
Александр Кудрявцев (Николаев)

Вопрос 21:
Группу женщин из фильма "Дюна" некий остряк назвал словосочетанием,
получившим известность два десятилетия назад. Напишите это
словосочетание.

Ответ:
(The) Spice Girls [зэ спайс гёрлз].

Комментарий:
В мире, созданном воображением фантаста Фрэнка Херберта, вещество под
названием "спайс" имеет огромную важность. Женская группа "Спайс Гёрлз"
была образована в 1994 году.

Источник:
   1. https://vk.com/frank_herberts_dune?w=wall-50406988_172
   2. http://ru.wikipedia.org/wiki/Пряность_(Дюна)
   3. http://ru.wikipedia.org/wiki/Spice_Girls

Автор:
Александр Кудрявцев (Николаев)

Вопрос 22:
Исследования одной южноафриканской пещеры обернулись для Андре КЕйсера
неожиданной находкой - ученый откопал человекообразное существо,
предположительно женского пола. Какое греческое имя получило это
существо?

Ответ:
Эвридика.

Комментарий:
Орфей отправился в подземное царство мертвых, чтобы вывести оттуда свою
Эвридику. Однако не совладал с собой и обернулся, нарушив поставленное
условие, из-за чего навсегда ее потерял.

Источник:
   1. http://www.istorion.ru/prehist/p_robustus.html
   2. http://ru.wikipedia.org/wiki/Эвридика

Автор:
Александр Кудрявцев (Николаев)

Вопрос 23:
Какая страна более двадцати лет находится под властью кровавого режима
ИсАйяса АфевОрки?

Ответ:
Эритрея.

Комментарий:
ИсАйяс АфевОрки - первый и единственный президент Эритреи, которая
провозгласила независимость в 1993 году. Название страны происходит от
греческого "&#7952;&#961;&#965;&#952;&#961;&#972;&#962;" [эрИтрос] -
"красный".

Источник:
   1. http://ru.wikipedia.org/wiki/Афеворк,_Исайяс
   2. http://www.ipress.ua/ljlive/erytreya_afrykanska_derzhava_shcho_bula_b_dorechnoyu_v_yevropi_foto_17810.html

Автор:
Александр Кудрявцев (Николаев)

Вопрос 24:
В басне Эзопа человек, купивший заморского раба, считает, что прежний
хозяин не заботился о нем, поскольку в буквальном смысле ДЕЛАЛ ЭТО.
Несмотря на проявленное усердие, труды нового хозяина пропадают впустую.
Какие слова мы заменили на "ДЕЛАТЬ ЭТО"?

Ответ:
Держать раба в черном теле.

Зачет:
Держать его в черном теле; держать в черном теле.

Комментарий:
В басне "Эфиоп" новый хозяин посчитал, что раба-африканца попросту
следует вымыть.

Источник:
http://www.deti-online.com/basni/basni-ezopa/yefiop/

Автор:
Александр Кудрявцев (Николаев)

Вопрос 25:
В 1978 году в Иордании неожиданно прервалась подходившая к завершению
трансляция "Евровидения", и лишь через некоторое время иорданским
телезрителям сообщили, что победил бельгийский певец Жан ВаллЕ. В какой
стране прошло "Евровидение" в 1979 году?

Ответ:
Израиль.

Комментарий:
Арабские страны враждебно относятся к Израилю. Трансляцию прервали,
когда стало ясно, что побеждает именно он. Как известно, местом
проведения "Евровидения" становится страна, победившая в минувшем году.

Источник:
http://www.israellycool.com/2014/05/09/israels-exit-from-eurovision-no-scandal/

Автор:
Александр Кудрявцев (Николаев)

Вопрос 26:
Герб заокеанской страны совсем не похож на герб наибольшей из ее
территорий. Тем не менее, крупное млекопитающее, изображенное на гербе
этой территории справа от щита, появилось там явно неслучайно. Назовите
это млекопитающее словом скандинавского происхождения.

Ответ:
Нарвал.

Комментарий:
Канада делится на провинции и территории. И на гербе Канады, и на гербе
территории НУнавут изображены единороги - мифический и вполне реальный.
В гербе Канады за основу взят герб Великобритании со львом и единорогом.
Нарвал и единорог - два названия одного и того же китообразного.

Источник:
   1. http://ru.wikipedia.org/wiki/Герб_Канады
   2. http://ru.wikipedia.org/wiki/Герб_Нунавута
   3. http://www.etymonline.com/index.php?term=narwhal&allowed_in_frame=0
   4. http://ru.wikipedia.org/wiki/Нарвал
   5. http://ru.wikipedia.org/wiki/Герб_Великобритании

Автор:
Александр Кудрявцев (Николаев)

Вопрос 27:
Объединитель Норвегии Харальд Косматый СДЕЛАЛ ЭТО после победы при
Хаврсфьорде. Назовите многодетного отца, который, согласно шутке,
получил свое прозвище за то, что не любил ДЕЛАТЬ ЭТО.

Ответ:
Всеволод [Юрьевич] Большое Гнездо.

Комментарий:
ДЕЛАТЬ ЭТО - расчесываться. Харальд Косматый дал слово не расчесываться,
пока не объединит Норвегию. Когда же это произошло, Харальд получил
новое прозвище - Прекрасноволосый. Всеволод Большое Гнездо в
действительности получил свое прозвище не за то, что не любил
расчесываться, а оттого, что имел много детей. Про растрепанного,
взлохмаченного человека говорят, что у него гнездо на голове.

Источник:
   1. http://ru.wikipedia.org/wiki/Харальд_I_Прекрасноволосый
   2. http://www.novosti-kosmonavtiki.ru/forum/messages/forum16/topic13192/message1025670/#message1025670
   3. http://ru.wikipedia.org/wiki/Всеволод_Юрьевич_Большое_Гнездо
   4. http://dic.academic.ru/dic.nsf/proverbs/17011/

Автор:
Александр Кудрявцев (Николаев)

Вопрос 28:
(pic: 20140515.jpg)
   Среди женщин самым высокооплачиваемым исполнительным директором
является МартИна РОтблатт, которая до 1994 года была ИМ. В экранизации
1955 года ЕМУ дарует свободу былой обидчик. Назовите этого обидчика.

Ответ:
Нильс [ХОльгерсон].

Комментарий:
ОН - Мартин. До 1994 года Мартина была Мартином, но сменила пол. В
мультфильме "Заколдованный мальчик" Нильс в начале попадает в гуся
Мартина камнем, а в конце выпускает из птичника.

Источник:
   1. http://www.meetup.com/Terasem/members/37506202/
   2. http://en.wikipedia.org/wiki/Martine_Rothblatt
   3. http://www.svoboda.org/content/article/26583241.html
   4. http://www.youtube.com/watch?v=CU5PsW3f9So

Автор:
Александр Кудрявцев (Николаев)

Вопрос 29:
Содержащийся в НЕЙ белок в шесть раз сильнее морфина, поэтому действие,
которое описывается устойчивым словосочетанием, имеет еще и
обезболивающий эффект. Тренер школьной команды из штата Орегон совершил
это действие на одной из тренировок, чтобы доказать, что он по-собачьи
предан своим подопечным. Назовите это действие двумя словами.

Ответ:
Зализывание ран.

Комментарий:
ОНА - слюна. Как оказалось, слюна обладает не только бактерицидными
свойствами. Тренер сказал, что готов даже зализывать раны своим
ученикам, после чего принялся вылизывать одному из них разбитое колено.

Источник:
   1. http://en.wikipedia.org/wiki/Opiorphin
   2. http://www.newsru.com/world/05aug2005/lick.html

Автор:
Александр Кудрявцев (Николаев)

Вопрос 30:
На лицевой стороне киргизской банкноты в сто сомов, имевшей хождение в
конце 1990-х, изображен народный акын ТоктогУл СатылгАнов. Какой хан
изображен на оборотной стороне этой банкноты?

Ответ:
Хан-ТЕнгри.

Зачет:
ТЕнгри; Небесный [хан]; [Хан] неба.

Комментарий:
Второй по высоте пик Тянь-Шаня. Обычно портреты выдающихся деятелей
размещают только на одной стороне банкноты, помещая на вторую
изображения природных объектов, архитектурных шедевров и т.п.

Источник:
   1. http://ru.wikipedia.org/wiki/Киргизский_сом
   2. http://ru.wikipedia.org/wiki/Сатылганов,_Токтогул
   3. http://ru.wikipedia.org/wiki/Киргизия

Автор:
Александр Кудрявцев (Николаев)

Вопрос 31:
Известный американец, предрекая повторный взлет популярности, сообщил
читателям своего блога, что их любимая жевательная резинка в 2016 году
снова войдет в моду. Коллега этого американца добавил, что спустя 25 лет
они обязаны дать ответы и на другие вопросы, не дававшие покоя
поклонникам. Назовите этого американца.

Ответ:
[Дэвид] Линч.

Комментарий:
Режиссер сообщил своим почитателям, что вскоре следует ожидать выхода на
экран продолжения сериала "Твин Пикс", и в 2016 году он вновь будет на
пике популярности. Ответ на самый знаменитый вопрос сериала, "Кто убил
Лору Палмер?", зрители узнали, но остались и другие вопросы.

Источник:
https://tjournal.ru/52588-twinpeaks-2016

Автор:
Александр Кудрявцев (Николаев)

Вопрос 32:
В замке ЛИндерхоф есть Грот Венеры, а в нем - искусственное озеро, для
плавания по которому построена лодка в форме АЛЬФЫ. Дизайнеры интерьеров
создали АЛЬФУ в форме АЛЬФЫ. Назовите АЛЬФУ.

Ответ:
Раковина.

Комментарий:
Венера появилась на свет из раковины. Лодка сделана в форме створки
раковины двустворчатого моллюска. Дизайнеры сделали кухонную раковину в
виде отпечатка спиральной раковины аммонита.

Источник:
   1. http://ru.wikipedia.org/wiki/Линдерхоф
   2. http://www.novate.ru/blogs/290608/9637/

Автор:
Александр Кудрявцев (Николаев)

Вопрос 33:
Глубинные гидротермальные источники являются своеобразными подводными
оазисами, около которых, по словам Дмитрия Писаренко, ПРОИСХОДИТ ЭТО.
Ответьте двумя словами, в которых все гласные одинаковы, что именно
происходит.

Ответ:
Кипит жизнь.

Комментарий:
Температура воды в черных курильщиках, о которых, собственно, и идет
речь, может превышать 400 градусов. Эти термальные источники привлекают
к себе большое количество глубоководных существ.

Источник:
   1. http://gazeta.aif.ru/_/online/aif/1183/15_02
   2. http://ru.wikipedia.org/wiki/Гидротермальные_источники_срединно-океанических_хребтов

Автор:
Александр Кудрявцев (Николаев)

Вопрос 34:
Словом "ИКС" мы заменили другое слово.
   Арт-акция Дилана Майера, завершившаяся убийством ИКСА, возмутила
жителей Сиэтла. Оправдываясь, Майер сказал, что схватка шла с переменным
успехом, и ИКС едва его не задушил. Жертвой ИКСА можно считать мужчину,
убитого 20 марта 1989 года. Назовите этого мужчину двумя или тремя
словами, начинающимися на одну и ту же букву.

Ответ:
Комиссар Коррадо Каттани.

Зачет:
Комиссар Каттани; Коррадо Каттани.

Комментарий:
ИКС - спрут. Дайвер Дилан Майер спустился под воду и убил гигантского
тихоокеанского спрута. "Спрут" - сериал об итальянской мафии.

Источник:
   1. http://www.npr.org/blogs/thetwo-way/2012/11/04/164287403/man-who-killed-octopus-for-art-project-angers-divers
   2. http://ru.wikipedia.org/wiki/Комиссар_Каттани

Автор:
Александр Кудрявцев (Николаев)

Вопрос 35:
На рождественской открытке трижды написано слово "Праздники", и в каждом
из этих слов выделены одни и те же две буквы. Напишите эти буквы.

Ответ:
H, o [эйч оу].

Комментарий:
На рождественской открытке трижды написано слово "Holidays" [хОлидейз].
Известно, что Санта смеется "Хо-хо-хо!".

Источник:
https://visualtextproject.wordpress.com/2011/12/24/visual-text-greeting/

Автор:
Александр Кудрявцев (Николаев)

Вопрос 36:
Александр Твардовский, принимавший участие в освобождении оккупированных
фашистами советских территорий, часто рассказывал историю о трехлетнем
мальчике, которого угостили кусочком хлеба. Какие два слова произнес
мальчик, взяв хлеб?

Ответ:
Danke Sch&ouml;n.

Зачет:
Данке шон; Данке шён.

Комментарий:
С рождения русский мальчик привык к немецкой речи.

Источник:
Документальный фильм "Три жизни поэта".

Автор:
Александр Кудрявцев (Николаев)

Тур:
Этап 3

Дата:
16-Nov-2014

Редактор:
Алексей и Мария Трефиловы (Калуга)

Инфо:
Редакторы благодарят за тестирование и ценные замечания: Баура
Бектемирова, Евгения Быстрова, Галину Воловник, Владимира Городецкого,
Людмилу Губаеву, Глеба Дрепина, Михаила Иванова, Дмитрия Когана, Николая
Константинова, Екатерину Лагуту, Михаила Малкина, Инну Семенову, Евгения
Миротина, Николая Рябых, Павла Столярова, Ивана Суманеева, Антона
Тахтарова, Александра Толесникова, Игоря Тюнькина.

Вопрос 1:
В конце XIX века АнтонИн Дворжак написал в Нью-Йорке симфонию, известную
также как "Симфония Нового света". Через 76 лет известный человек взял
запись этой симфонии с собой. Назовите этого человека или любого из его
спутников.

Ответ:
[Нил] Армстронг.

Зачет:
[Майкл] Коллинз, [Эдвин/Базз] Олдрин.

Комментарий:
Символично, что эту симфонию Армстронг взял с собой в полет на Луну. Из
одного Нового света - к другому.

Источник:
http://ru.wikipedia.org/wiki/Симфония_%E2%84%96_9_(Дворжак)

Автор:
Игорь Тюнькин (Москва)

Вопрос 2:
Индейский герой Тийо получил от НЕЕ в подарок плетеную корзину. Другая
ОНА, появившаяся в тридцать втором выпуске в феврале 1977 года, была
дочерью биологов. Назовите ЕЕ.

Ответ:
Женщина-паук.

Зачет:
Spider-Woman. Незачет: Паучиха.

Комментарий:
То, что корзина плетеная - намекает на паутину. Во втором случае - речь
про героиню комиксов.

Источник:
   1. Spider Woman Stories: Legends of the Hopi Indians, p. 32.
   2. http://en.wikipedia.org/wiki/Spider-Woman

Автор:
Баур Бектемиров (Алматы)

Вопрос 3:
В абсурдистском романе Бориса Виана повар применил к НЕМУ
стоматологические щипцы. Рассказывают, что муж одной актрисы давал ЕГО
своей жене перед съемками. Назовите ЕГО.

Ответ:
Чеснок.

Зачет:
Зубчик чеснока.

Комментарий:
Ревнивый муж давал жене есть чеснок, если по сценарию предполагалась
сцена с поцелуем. В романе Виана повар выдирал зубчик чеснока щипцами.

Источник:
   1. http://lib.ru/WIAN/pena0.txt
   2. Легенды мирового кино. Грегори Пек.

Автор:
Мария Трефилова (Калуга), Игорь Тюнькин (Москва)

Вопрос 4:
Ирина Можаева рассказывает, что нередко ЭТО было частью приданого, так
как служило напоминанием невесте, уходящей в новый дом, о каждом из ее
родственников. Назовите ЭТО двумя словами.

Ответ:
Лоскутное одеяло.

Комментарий:
Это одеяло часто шили из лоскутков тканей, использовавшихся для пошива
одежды других членов семьи.

Источник:
http://www.artrussian.com/vopros_72.html

Автор:
Алексей Трефилов (Калуга)

Вопрос 5:
[Ведущему: не сообщать командам, что слово "Родина" взято в кавычки.]
   В советское время в бухте закрытого военного городка Балаклава на
одном из кораблей иногда натягивали полотно. И тогда местные жители о
"Родине" уже не вспоминали. Сам этот корабль они называли плавучим...
Чем?

Ответ:
Кинотеатром.

Зачет:
Кинотеатр.

Комментарий:
Морякам показывали кино, и гражданским тоже было хорошо видно, поэтому в
кинотеатр "Родина" в эти вечера они не шли.

Источник:
"Остров Крым". Серия про Балаклаву.

Автор:
Алексей Трефилов (Калуга)

Вопрос 6:
(pic: 20140516.jpg)
   Эта модель обуви названа в честь НЕГО. В честь НЕГО назван и мост в
окрестностях Рима. Назовите ЕГО.

Ответ:
[Абебе] БикИла.

Комментарий:
Эта минималистичная модель спортивной обуви позволяет стопе чувствовать
себя естественно. Как известно, Абебе БикИла в 1960 году выиграл
"золото" в марафоне Римской олимпиады босиком.

Источник:
http://en.wikipedia.org/wiki/Abebe_Bikila

Автор:
Алексей Трефилов (Калуга)

Вопрос 7:
В романе Роберта Ирвина персонаж-писатель расстался с любимой. Так
сказать, любовная лодка разбилась о быт. Спустя несколько лет он захотел
ее разыскать. И для этого решил издать автобиографическую книгу.
Назовите тремя словами то, с чем он сравнил эту книгу.

Ответ:
Письмо в бутылке.

Зачет:
Послание в бутылке.

Комментарий:
Герой не знал, как разыскать бывшую возлюбленную, и рассчитывал, что она
прочтет его книгу и откликнется. Хотя и понимал, что шансов немного.
Метафора Маяковского про "любовную лодку" - намек на морскую тематику.

Источник:
http://www.e-libra.ru/read/222591-utonchennyj-mertvec-exquisite-corpse.html

Автор:
Мария Трефилова, Алексей Трефилов (Калуга)

Вопрос 8:
На одной из своих картин Рене Магритт выразил отношение к женщине как к
немому объекту плотских утех и изобразил, так сказать, ЕЕ наоборот.
Назовите ЕЕ.

Ответ:
Русалка.

Комментарий:
(pic: 20140517.jpg)
   Изобразил немую верхнюю половину рыбы и нижнюю обнаженную половину
женщины.

Источник:
Гении и злодеи. Рене Магритт.

Автор:
Игорь Тюнькин (Москва)

Вопрос 9:
Однажды поэт Евгений Винокуров сделал заказ в ресторане. Но потом
окликнул уходящего официанта и заказал один кофе. А что он заказал
сначала?

Ответ:
Одно кофе.

Комментарий:
Винокуров решил, что поэту не следует быть небрежным в языке, и
исправился.

Источник:
http://www.poezia.ru/salon.php?sid=21773

Автор:
Алексей Трефилов (Калуга)

Вопрос 10:
Леонардо да Винчи считал, что у НЕЕ нет органов пищеварения. Раньше в
Англии ЕЮ называли кочергу. Назовите ЕЕ.

Ответ:
Саламандра.

Комментарий:
В старину считали, что саламандра питается от огня через кожу. Кочерга,
как и саламандра, тоже часто оказывалась в очаге.

Источник:
   1. http://en.wikipedia.org/wiki/Salamanders_in_folklore_and_legend
   2. http://en.wiktionary.org/wiki/salamander

Автор:
Баур Бектемиров (Алматы)

Вопрос 11:
Внимание, дуплет! Два вопроса по 30 секунд обсуждения на каждый.
Напишите сначала один ответ, и рядом - второй.
   1. В одном хокку ОНА сыпала жемчуг в изумруд. Назовите ЕЕ.
   2. На иллюстрации учебника естественной истории 1864 года изображены
два орангутана. Что держит в руке меньший из них?

Ответ:
   1. Яблоня.
   2. Яблоко.

Комментарий:
В учебнике - орангутаны в роли Адама и Евы. Преамбула к вопросу
намекала, что в ваших ответных карточках "яблоко" окажется недалеко от
"яблони". В стихотворении так описывается опадающий цвет - лепестки
яблони белые и округлые. Но если вы подумали, что речь про маленькие
яблочки, то тоже хорошо. :-)

Источник:
   1. http://www.ordenknights.ru/l_ballad/ballad20.php
   2. Boria Sax. Imaginary animals, p. 48.

Автор:
Алексей Трефилов (Калуга), Баур Бектемиров (Алматы)

Вопрос 12:
Раньше в продукции комбината, выпускающего киноматериалы, попадалось
немало брака, поэтому операторы, заряжая пленку, по словам Эдуарда
Тополя, ДЕЛАЛИ ЭТО. Ответьте четырьмя словами: что делали?

Ответ:
Играли в русскую рулетку.

Комментарий:
Операторы рисковали, ведь значительная часть работы могла оказаться
напрасной. "Заряжая пленку" - подсказка. Кроме того, вращающиеся бобины
с пленкой могут напомнить барабан револьвера.

Источник:
"Story", 2014, N 10. - С. 72.

Автор:
Мария Трефилова, Алексей Трефилов (Калуга)

Вопрос 13:
По словам Анны Север, поэтесса Серебряного века ИраИда ГУставовна
ГЕйнике стала Ириной Владимировной ОдОевцевой, сменив имя практически
одновременно с НИМ. Назовите ЕГО.

Ответ:
Санкт-Петербург.

Зачет:
Петербург, Петроград, Питер. Незачет: Ленинград.

Комментарий:
В связи с началом Первой мировой войны и неприятием всего немецкого
имена меняли люди и города: Санкт-Петербург стал Петроградом.

Источник:
"Story", 2014, N 8. - С. 58.

Автор:
Мария Трефилова (Калуга)

Вопрос 14:
Расположенная на побережье Балтийского моря КУршская коса долгое время
страдала от наступающих песчаных дюн. Из-за нехватки плодородных земель
местные жители были вынуждены питаться в основном рыбой. И, чтобы
разнообразить рацион, они околачивали груши, расставляя сети на берегу.
Восстановите два слова, которые мы заменили в тексте вопроса.

Ответ:
Ловили ворон.

Зачет:
Ловили галок. Незачет: Считали ворон.

Комментарий:
Леса, сдерживающие песчаные дюны, были сильно вырублены. Было сложно
найти другую дичь. Да и выращивать сельхозкультуры рядом с домом также
было проблематично. Фразеологизм "ловить ворон" употребляется наряду с
фразеологизмом "считать ворон".

Источник:
Д/ф "Куршская коса". Эфир RTG.

Автор:
Алексей Трефилов (Калуга)

Вопрос 15:
(pic: 20140518.jpg)
   АкутагАва РюнОскэ писал про своего остроумного персонажа, что тот
применял два ИКСА: иронию и юмор. Назовите ИКС.

Ответ:
[Самурайский] меч.

Комментарий:
Раздаточный материал намекает на пару самурайских мечей, лежащих на
традиционной подставке. Слово "остроумный" и то, что Акутагава - японец,
тоже может помочь в раскрутке. :-)

Источник:
http://www.litmir.net/br/?b=221009&p=70

Автор:
Баур Бектемиров (Алматы)

Вопрос 16:
В историческом романе описывается, как этого человека, помещенного в
темницу за вольнодумство, в наказание выводили на прогулку только ночью.
Александр Казанцев замечает, что этот человек взял себе имя словно в
честь английского коллеги-предшественника. Назовите этого человека.

Ответ:
[Томмазо] Кампанелла.

Комментарий:
Автора утопии "Город Солнца" лишали солнечного света. Томмазо - это имя,
под которым Кампанелла был пострижен в монахи. Томас Мор - другой
известный утопист.

Источник:
http://lib.ru/RUFANT/KAZANCEW/sunkolokol.txt

Автор:
Алексей Трефилов (Калуга)

Вопрос 17:
Джотто был новатором и, изображая ИКС святого, делал нимб овальным.
Какое слово мы заменили на ИКС?

Ответ:
Профиль.

Комментарий:
Джотто таким образом старался передать перспективу. Он решил, что при
повороте головы нимб тоже поворачивается и визуально выглядит иначе.

Источник:
Цикл "Энциклопедия". Джотто.

Автор:
Алексей Трефилов (Калуга)

Вопрос 18:
В российском документальном фильме говорится, что дома местных рыбаков
имели некоторые архитектурные особенности. Например, входные двери
состояли из двух створок: нижней и верхней, чтобы после очень ветреной
ночи можно было открыть хотя бы верхнюю. Этот фильм рассказывает о НЕЙ.
Назовите ЕЕ двумя словами, начинающимися на одну и ту же букву.

Ответ:
Куршская коса.

Комментарий:
Как вы уже знаете, местные жители на Куршской косе страдали от
наступающих дюн. Пески засыпАли деревни. И жители шли на хитрость,
позволяющую легко выбраться из засыпанного дома.

Источник:
Д/ф "Куршская коса". Эфир RTG.

Автор:
Алексей Трефилов (Калуга)

Вопрос 19:
Записавшись на курсы в Манчестере, жительница Белфаста Диана ЛИндблатт
доехала поездом до Дублина, оттуда паромом до ХОлихеда, а затем еще
четыре часа добиралась до Манчестера. У всех слушателей этих курсов была
ОНА. Назовите ЕЕ одним словом.

Ответ:
Аэрофобия.

Комментарий:
Диана записалась на специальные курсы, чтобы избавиться от аэрофобии, и,
так как она боялась полетов, ей пришлось проделать столь долгий путь.

Источник:
BBC. "Страх".

Автор:
Игорь Тюнькин (Москва)

Вопрос 20:
Бывший разведчик Михаил Любимов иронично замечает, что распознать
предателя непросто. Ведь когда видишь этого человека, то нет никакого
навевающего тревогу и подозрения ЕГО. Назовите ЕГО словом английского
происхождения.

Ответ:
Саундтрек.

Зачет:
Soundtrack.

Комментарий:
Любимов иронизирует, что при просмотре шпионских фильмов разбираться в
людях гораздо проще.

Источник:
"Story", 2014, N 5. - С. 33.

Автор:
Мария Трефилова (Калуга)

Вопрос 21:
[Ведущему: очень важно читать стихотворение очень медленно, под запись,
делая значительные паузы в конце строк!]
   Послушайте танка поэтессы ЁсАно АкИко:
   Белеют плечи мои -
   В деревянной бадье купаюсь;
   Некто подумает, увидав:
   Белых кувшинок бутоны
   Распустились.
   Комментируя это стихотворение, Елена Дьяконова отмечает, что ЁсАно
свойственен ОН. Назовите ЕГО.

Ответ:
Нарциссизм.

Комментарий:
Ёсано в стихотворении открыто любуется собой, что очень нехарактерно для
традиционной японской поэзии. При этом она сравнивает себя с цветком.
Вода, цветы, самолюбование - все атрибуты налицо. :-)

Источник:
Ёсано Акико. Спутанные волосы. - М.: Эксмо, 2007. - С. 26-27.

Автор:
Алексей Трефилов (Калуга)

Вопрос 22:
Персонаж игры "Ведьмак 2" считает, что люди и эльфы конфликтуют из-за
пустяка. Поэтому некие особенности персонаж описывает с помощью двух
прилагательных. Напишите эти прилагательные.

Ответ:
Тупоконечный, остроконечный.

Зачет:
В любом порядке.

Комментарий:
"Война из-за того, остроконечные или тупоконечные у вас уши, так же
нелепа, как и конфликт из-за стороны разбивания яйца". Такая вот отсылка
к персонажам Свифта.

Источник:
Компьютерная игра "Ведьмак 2. Убийца королей".

Автор:
Игорь Тюнькин (Москва)

Вопрос 23:
Эрих фон Деникен считает, что древние люди на протяжении некоторого
времени общались с пришельцами, пока те не улетели. Фон Деникен
полагает, что мифы о богах и объекты, такие как в пустыне НАска, - это
отголоски своеобразного ЕГО. Термин "ОН" впервые был упомянут в печати в
1945 году. Напишите этот термин.

Ответ:
Культ карго.

Зачет:
Карго-культ.

Комментарий:
Широкую известность термин "культ карго" получил после Второй мировой
войны. Американские войска покинули ряд тихоокеанских островов, а
местные жители, привыкшие к тому, что американцы снабжали их едой, стали
строить копии взлетно-посадочных полос и самолетов из кокосовых пальм и
соломы. Они верили, что эти постройки привлекут заполненные грузами
самолеты, считавшиеся посланниками духов.

Источник:
   1. Д/ф "Новые "Воспоминания о будущем"".
   2. http://en.wikipedia.org/wiki/Cargo_cult

Автор:
Алексей Трефилов (Калуга)

Вопрос 24:
В сказке Петра МордкОвича демиурги разбирают хрустальный купол и
нечаянно раскалывают одну из панелей. После чего один спрашивает
другого, успел ли он СДЕЛАТЬ ЭТО. Что сделать?

Ответ:
Загадать желание.

Комментарий:
Осколки хрустального купола неба - это падающие звезды. Демиурги в этой
сказке убирали хрустальный купол от геоцентрической системы и собирались
устанавливать систему гелиоцентрическую. :-)

Источник:
http://bormor.livejournal.com/171643.html

Автор:
Алексей Трефилов (Калуга)

Вопрос 25:
Хельмут НьЮтон рассказывает, что в 60-х годах в Сен-Тропе жандармерия
распыляла с вертолетов несмываемую краску над НИМИ. Над кем?

Ответ:
Над нудистами.

Зачет:
Над натуристами.

Комментарий:
Чтобы потом найти и оштрафовать. Кстати, в старой французской комедии
про жандармов из Сен-Тропе тоже охотились на нудистов. :-)

Источник:
"Story", 2014, N 5. - С. 120.

Автор:
Мария Трефилова (Калуга)

Вопрос 26:
В рассказе Евгения Замятина персонаж, отвечая на вопрос, выгнул грудь
колесом, выставил ногу вперед и сказал: "Председатель". Какое слово мы
пропустили в тексте вопроса?

Ответ:
Я.

Комментарий:
Отвечая на вопрос, кто в деревне председатель, персонаж принял позу,
напоминающую букву "Я" и сказал: "Председатель - я". Как известно,
Замятин - автор романа "Мы".

Источник:
http://az.lib.ru/z/zamjatin_e_i/text_0150.shtml

Автор:
Мария Трефилова (Калуга)

Вопрос 27:
Екатерина ЧунькОва рассказывает, что кабинет Пушкина в усадьбе
Михайловское воссоздан достаточно точно. Только теперь здесь появился
ИКС, какого не было при жизни Пушкина. ИКСОМ мы заменили слово из семи
букв. Напишите их в правильной последовательности.

Ответ:
Порядок.

Зачет:
П, о, р, я, д, о, к.

Комментарий:
В кабинете Пушкина всегда был творческий беспорядок. Мы попросили
написать буквы в правильном порядке.

Источник:
Д/ф "Пушкинские горы". Эфир RTG.

Автор:
Алексей Трефилов (Калуга)

Вопрос 28:
В старые времена, чтобы добыть ЕГО, использовали багры и сачки. Среди
его разновидностей выделяют "капли", "натёки" и "сосульки". Назовите
ЕГО.

Ответ:
Янтарь.

Комментарий:
В Прибалтике багром рыхлили морское дно неподалеку от берега или
ворошили водоросли, легкий янтарь всплывал на поверхность, и его
собирали сачком. Янтарь нередко различают по тем формам, которые
принимала смола при застывании.

Источник:
   1. Д/ф "Тайны янтаря". Эфир телеканала "Культура".
   2. http://www.nkj.ru/archive/articles/9199/

Автор:
Алексей Трефилов (Калуга)

Вопрос 29:
(pic: 20140519.jpg)
   В этой работе фотохудожник Патрик Гонсалез хотел показать
мимолетность молодости, поэтому девушка на портрете держит ЕГО. Назовите
ЕГО.

Ответ:
Одуванчик.

Комментарий:
Молодость так же мимолетна и скоротечна, как цветение одуванчика. На
портрете образ девушки как будто сдувается ветром, как пух одуванчика в
ее руках. На это также намекают пушинки около ее руки и около ее головы.
Слово "мимолетность" - дополнительная подсказка.

Источник:
http://patrickgonzales.35photo.ru/photo_396827/

Автор:
Мария Трефилова (Калуга)

Вопрос 30:
Супруга Франца Иосифа Елизавета Австрийская была вынуждена отказаться от
одного своего увлечения. У нее появились проблемы с позвоночником из-за
той разновидности, которую она использовала. Назовите эту разновидность
двумя словами.

Ответ:
Дамское седло.

Зачет:
Женское седло.

Комментарий:
Елизавета увлекалась верховой ездой. Дамское седло, когда сидят на
лошади боком, предполагает скручивание позвоночника.

Источник:
"Story", 2013, N 3. - С. 133.

Автор:
Мария Трефилова (Калуга)

Вопрос 31:
Валентин Берестов в своем стихотворении говорит о том, что усилия, даже
кажущиеся напрасными, могут дать результат. Цитата:
   Дружно ударились [пропуск 1],
   И начался на реке [пропуск 2].
   Заполните первый пропуск тремя словами.

Ответ:
"... рыбы об лед...".

Комментарий:
Фразеологизм "биться как рыба об лед" означает "бесплодно чего-то
добиваться", но поэт его переосмыслил. Полностью стихотворение звучит
так:
   Дружно ударились рыбы об лед,
   И начался на реке ледоход.

Источник:
http://www.poesis.ru/poeti-poezia/berestov/frm_vers.htm

Автор:
Алексей Трефилов (Калуга)

Вопрос 32:
По легенде, рыцарь ПаццИно де ПАцци за доблесть при взятии Иерусалима
получил камни Храма Гроба Господня и к некоему дню доставил их во
Флоренцию. В тот день ПаццИно с помощью этих камней получил... Ответьте
максимально точно, что именно.

Ответ:
Благодатный огонь.

Зачет:
Святой свет; святой огонь; пасхальный огонь; священный огонь; огонь
[Храма] Гроба Господня.

Комментарий:
Эти камни Паццино доставил к Пасхе и высек с их помощью Благодатный
огонь. Обычно этот огонь выносят из Гроба Господня на богослужении
накануне Пасхи.

Источник:
   1. http://ru.toskana-netz.de/174/2010-0221/scoppio-del-carro-2010.html
   2. http://en.wikipedia.org/wiki/Scoppio_del_carro
   3. http://www.duomofirenze.it/feste/pasqua_eng.htm
   4. http://ru.wikipedia.org/wiki/Благодатный_огонь

Автор:
Алексей Трефилов (Калуга)

Вопрос 33:
Преследуя человека на темной лестнице, герой Достоевского почувствовал,
как пару раз по ПЕРВОМУ ударил край ВТОРОЙ. Назовите ПЕРВЫЙ и ВТОРУЮ.

Ответ:
Нос, шинель.

Комментарий:
А до того герой преследовал человека на Невском проспекте. Достоевский -
славный продолжатель дела Гоголя в написании "петербургских повестей".
:-) Очевидно, что в эпизоде отсылка к названиям повестей Гоголя сделана
специально.

Источник:
http://az.lib.ru/d/dostoewskij_f_m/text_0140.shtml

Автор:
Алексей Трефилов (Калуга)

Вопрос 34:
Знаете, что меня всегда удивляло в английском языке? Все эти слова с
кучей разных значений. Вот попадется вам слово "boobs" [бубз] - не
спешите радоваться. Возможно, речь не про женскую грудь. А про чьи-то
"досадные ошибки". Или, скажем, репортаж в новостях. Когда журналист
берет микрофон, чтобы сказать в кадре что-то несмешное. И как такой
прием называется?

Ответ:
Стендап.

Зачет:
Стэндап; стенд-ап; стэнд-ап; stand up; standup; stand-up.

Комментарий:
Репортерский прием, когда журналист работает непосредственно в кадре на
месте событий, тоже называется "стендап". Текст вопроса стилизован под
монолог стендап-комика. Надеемся, вас это немного позабавило. :-)

Источник:
   1. http://ru.wikipedia.org/wiki/Стендап
   2. http://www.ehow.com/how_4457601_do-stand-up-tv-news.html

Автор:
Алексей Трефилов, Мария Трефилова (Калуга)

Вопрос 35:
Во время московского кинофестиваля Джина Лоллобриджида и Элизабет Тейлор
улыбались и всё время держались вместе. Однако через некоторое время ОН
получил два судебных иска. Напишите его фамилию, использовав дефис.

Ответ:
Сен-Лоран.

Комментарий:
Так получилось, что у актрис оказались два одинаковых платья от одного
модельного агентства. Когда это выяснилось, Элизабет и Джина решили
выйти из положения, держась всё время вместе. Словно так и было
задумано.

Источник:
Легенды мирового кино. Джина Лоллобриджида.

Автор:
Игорь Тюнькин (Москва)

Вопрос 36:
В абхазской сказке рассказчик, упоминая ИХ, предлагает убедиться: вот,
смотрите, до сих пор еще мокрые. Назовите ИХ.

Ответ:
Усы.

Комментарий:
Это присказка в конце сказки. В абхазском варианте ее начало звучит так:
"И я на том пиру был, усы в вине мочил...".

Источник:
Сказки народов Закавказья. - Цхинвали: Ирыстон, 1987. - С. 14.

Автор:
Алексей Трефилов (Калуга)

Тур:
Этап 4

Дата:
14-Dec-2014

Редактор:
Дмитрий Великов (Москва), Александр Рождествин (Самара), Сергей Спешков
(Москва)

Инфо:
Редакторы благодарят за помощь в подготовке пакета команды "Биркиркара",
"Гросс-бух" и "Эрудиты" (все - Москва), а также Антона Тахтарова и Елену
Конькову (оба - Самара), Евгения Миротина (Минск), Николая Рябых
(Королёв), Александра Маркова, Ивана Семушина, Екатерину и Станислава
Мереминских, Романа Немучинского, Владимира Цвингли, Сергея Даровских
(все - Москва), Ивана Ефремова (Ростов-на-Дону), Константина Сахарова
(Ивантеевка), Дмитрия Когана (Фридрихсхафен), Александра Митрякова
(Ижевск), Александра Коробейникова (Саратов).

Вопрос 1:
Провожая Мари в волшебную страну, Щелкунчик распахивает перед нею дверь
ИКСА. Назовите ИКС двумя словами.

Ответ:
Платяной шкаф.

Зачет:
Волшебный шкаф.

Комментарий:
Позже подобным путем дети попадали в Нарнию.

Источник:
Э.Т.А. Гофман, Щелкунчик и мышиный король.
http://www.deti-online.com/skazki/skazki-gofmana/shelkunchik-i-myshinyi-korol/

Автор:
Дмитрий Великов (Москва)

Вопрос 2:
Персонаж фильма Дэвида Финчера в русском переводе говорит, что чувствует
себя боксерской грушей, из которой пытаются выбить побольше денег. Что
персонаж упоминает в этом месте в оригинале?

Ответ:
Пиньяту.

Комментарий:
Видимо, переводчики решили избавиться от малоизвестной реалии.

Источник:
Фильм "Игра" (1997).

Автор:
Сергей Спешков (Москва)

Вопрос 3:
Читатели недавно опубликованного романа обратили внимание на подробное
описание некоторых деталей женского гардероба, после чего продажи книги
сильно выросли. Назовите автора этого романа.

Ответ:
[Роберт] Гэлбрейт.

Зачет:
[Джоан] Роулинг.

Комментарий:
Детектив "Зов кукушки" вышел под именем Роберта Гэлбрейта, но
внимательные читатели быстро вычислили, что под этим псевдонимом
скрывается Джоан Роулинг.

Источник:
http://lenta.ru/news/2013/07/14/rowling/

Автор:
Александр Рождествин (Самара)

Вопрос 4:
Внимание, в вопросе есть замена.
   Во время Второй мировой войны ИКСЫ делали из серебра, меди и
марганца, потому что весь ИКС отправлялся на военные нужды. Назовите
ИКС.

Ответ:
Никель.

Комментарий:
Никель - обиходное название американской монеты в пять центов. Так
называемые "военные никели" никакого никеля на самом деле не содержали.
Кстати, и сейчас они делаются из сплава, в котором три четверти - медь.

Источник:
http://en.wikipedia.org/wiki/Nickel_(United_States_coin)

Автор:
Александр Рождествин (Самара)

Вопрос 5:
Со временем ОНА стала утрачивать первоначальное назначение, и в
правление губернатора ЛАклана МаккуОри для аналогичных целей стали всё
чаще использовать объект, находящийся еще южнее. Назовите этот объект.

Ответ:
[Остров] Тасмания.

Зачет:
[Остров] Земля Ван-ДИмена.

Комментарий:
Австралия поначалу была местом ссылки преступников. Когда заканчивался
срок каторги, они чаще всего оставались свободными поселенцами. А для
рецидивистов и редких новых партий каторжников стали использовать
Тасманию. Фактически Тасмания для Австралии стала тем, чем раньше была
Австралия для Британии.

Источник:
   1. http://en.wikipedia.org/wiki/Tasmania
   2. http://en.wikipedia.org/wiki/Lachlan_Macquarie
   3. http://en.wikipedia.org/wiki/New_South_Wales

Автор:
Сергей Спешков (Москва)

Вопрос 6:
В рекламном ролике одного аукционного дома люди отчаянно выкрикивают
цену каждый со своего места, а в это время под ними перемещаются
"устроители" аукциона с НИМ в руках. Назовите ЕГО.

Ответ:
[Пожарный] тент.

Зачет:
[Пожарный] брезент, [пожарный] батут.

Комментарий:
Дело происходит на пожаре. Каждый невольный "участник" аукциона из
своего окна предлагает цену, а пожарные внизу мечутся от одного к
другому.

Источник:
http://www.youtube.com/watch?v=fvjGKZNT1vQ

Автор:
Сергей Спешков (Москва)

Вопрос 7:
Художник Альберт БИрштадт серьезно подходил к выбору вида для своих
пейзажей. В статье русскоязычной википедии "БИрштадт" присутствует число
4287. Какое слово мы пропустили в предыдущем предложении?

Ответ:
Гора.

Зачет:
Вершина, пик.

Комментарий:
Речь о статье Википедии "БИрштадт (гора)", которую покорил художник в
поисках вдохновения. Гора впоследствии была названа в его честь.
Впрочем, в итоге картины он написал только у подножия горы.

Источник:
   1. http://www.14ers.com/php14ers/historyview.php?parmpeak=Mt.%20Bierstadt&parmcat=Name%20History
   2. http://en.wikipedia.org/wiki/Albert_Bierstadt
   3. http://ru.wikipedia.org/wiki/Бирштадт_(гора)

Автор:
Александр Рождествин (Самара)

Вопрос 8:
Среди причин поражения сипаев в восстании в середине девятнадцатого века
было техническое превосходство британцев не только в вооружении. Один из
приговоренных к смертной казни мятежников назвал ЕГО "проклятой
веревкой, которая душит". Назовите ЕГО, использовав слово греческого
происхождения.

Ответ:
Телеграфный провод.

Зачет:
Телеграф, телеграфный кабель.

Комментарий:
Благодаря новому для того времени средству связи британцы могли гораздо
лучше и быстрее координировать свои действия.

Источник:
Ниал Фергюсон. Империя. Чем современный мир обязан Британии.
http://www.flibusta.net/b/378579/read

Автор:
Сергей Спешков (Москва)

Вопрос 9:
Постоянно курящий герой Джозефа Хеллера носит фамилию ИКС. В африканском
фольклоре ИКСОВ уважают за то, что те не боятся солдат. Какое слово в
этом вопросе мы заменили на ИКС?

Ответ:
Трубкозуб.

Зачет:
Аардваарк, Aardvaark.

Комментарий:
У героя Хеллера в зубах постоянно находится курительная трубка.
Трубкозубы питаются термитами и муравьями и не боятся муравьев-солдат.
Впрочем, толстая кожа хорошо защищает от их укусов.

Источник:
   1. http://lib.ru/INPROZ/HELLER/catch22.txt
   2. http://www.themagicalbuffet.com/Issues/Vol02_Iss07/Article_049.html
   3. http://ru.wikipedia.org/wiki/Трубкозуб

Автор:
Сергей Спешков (Москва)

Вопрос 10:
В 1815 году герцог Ришелье вернулся во Францию, для того чтобы занять
пост премьер-министра. Говоря об этом событии, Леонид Парфёнов вспомнил
известную фразу, в которой последнее слово заменил топонимом.
Воспроизведите получившуюся фразу.

Ответ:
Париж стоит Одессы.

Комментарий:
Дюк Ришелье вернулся в Париж из России, оставив должность
генерал-губернатора Новороссийского края и одесского градоначальника.

Источник:
   1. http://en.wikipedia.org/wiki/Armand-Emmanuel_de_Vignerot_du_Plessis,_Duc_de_Richelieu
   2. http://www.youtube.com/watch?v=OXnJPzGSWhk&t=38m48s

Автор:
Александр Рождествин (Самара)

Вопрос 11:
В одной компьютерной игре можно найти могилу некоего Марка. Эпитафия
гласит: "Он был полон великих идей, но не работал ни дня". Напишите
четырехбуквенную фамилию этого Марка.

Ответ:
Сизм.

Зачет:
Сист.

Комментарий:
(pic: 20140520.jpg)
   Это своего рода шутка от разработчиков, которые не очень-то верят в
марксизм.

Источник:
Игра "Divinity: Original sin".

Автор:
Александр Рождествин (Самара)

Вопрос 12:
Описывая ощущения от разговора с Антеем, Данте упоминает болонскую
АЛЬФУ. Возможно, это связано с тем, что другая АЛЬФА к тому времени была
еще не закончена. Какое слово мы заменили словом "АЛЬФА"?

Ответ:
Башня.

Комментарий:
Склонившегося к нему гиганта Данте сравнивает с Гаризендой - падающей
башней, находящейся в Болонье. Самая известная падающая башня -
пизанская.

Источник:
   1. Данте Алигьери. Божественная комедия.
http://www.infoliolib.info/flit/dante/ad31.html
   2. http://ru.wikipedia.org/wiki/Пизанская_башня

Автор:
Александр Рождествин (Самара)

Вопрос 13:
В стихотворении Николая Заболоцкого "Стирка белья" ОНА появляется из
корыта. Назовите ЕЕ.

Ответ:
Афродита.

Зачет:
Венера.

Комментарий:
Морскую пену и раковину у поэта, по-видимому, символизируют мыльная пена
и корыто.

Источник:
http://rupoem.ru/zabolockij/v-storone-ot.aspx

Автор:
Сергей Спешков (Москва)

Вопрос 14:
Кинокомпании не хотели снимать этот фильм, поскольку ранее в прокате
провалилась картина "Братство". Автор сценария этого фильма на встрече с
продюсером сказал: "Я задолжал одиннадцать тысяч, если не верну, мне
сломают руку". Назовите автора сценария.

Ответ:
[Марио] Пьюзо.

Комментарий:
Речь идет о сценарии фильма "Крестный отец". Поскольку другой фильм об
организованной преступности незадолго до того провалился, снимать новый
компании не решались.

Источник:
П. Бискинд. Беспечные ездоки, бешеные быки.
http://www.flibusta.net/b/240883/read

Автор:
Сергей Спешков (Москва)

Вопрос 15:
Византийский историк Анна КомнинА, вспоминая времена могущества Римской
империи, упоминает еще и ДионИсовы на востоке. Какое слово мы пропустили
в этом вопросе?

Ответ:
Столпы.

Зачет:
Столбы.

Комментарий:
Анна пишет, что империя простиралась от столпов Геракла на западе до
столпов Диониса на востоке. Впрочем, современные историки не в состоянии
объяснить, что именно Анна понимала под столпами Диониса.

Источник:
Анна Комнина. Алексиада. http://www.flibusta.net/b/28096/read

Автор:
Александр Рождествин (Самара)

Вопрос 16:
Первая часть термина "сахамарАна" переводится как "вместе". Первая часть
термина "анумарАна" - как "после". Разновидностями чего являлись
сахамарАна и анумарАна?

Ответ:
САти.

Зачет:
[Само]сожжения вдовы.

Комментарий:
В Индии жены умерших мужей могли сжигаться либо вместе с телом, либо
после. Это определялось в зависимости от определенных условий.

Источник:
   1. http://en.wikipedia.org/wiki/Sati_(practice)#Anumarana
   2. http://en.wikipedia.org/wiki/Anumarana

Автор:
Сергей Спешков (Москва)

Вопрос 17:
Джеймс Джойс описывает, как змеи увиваются вокруг посоха. Что потом
делают из чешуи этих змей?

Ответ:
Пиво.

Комментарий:
Так Джойс описывает хмель, который является одним из основных
ингредиентов при производстве пива. Хмель - вьющееся растение, шишки
которого напоминают чешую.

Источник:
Джеймс Джойс. Улисс.

Автор:
Сергей Спешков (Москва)

Вопрос 18:
ДЕян СУджич иронично назвал его богом домашнего очага. Какое
изобретение, по словам СУджича, избавило людей от необходимости
регулярного ритуального поклонения этому "богу"?

Ответ:
Пульт дистанционного управления.

Зачет:
ПДУ, пульт от телевизора, прочие эквивалентные ответы со словом "пульт"
без неверных уточнений.

Комментарий:
Суджич назвал богом домашнего очага телевизор. До изобретения ПДУ для
включения телевизора и переключения каналов нужно было подходить к
телевизору и буквально склоняться перед ним.

Источник:
Деян Суджич. Язык вещей. http://www.flibusta.net/b/339479/read

Автор:
Сергей Спешков (Москва)

Вопрос 19:
Слово "АЛЬФА" в вопросе - замена.
   У аристократа Герберта Китченера было плохое зрение. Поэтому Китченер
с юмором использовал в качестве АЛЬФ Выстрел, Промах и Проклятье. Какая
АЛЬФА стала названием произведения девятнадцатого века?

Ответ:
Муму.

Комментарий:
АЛЬФА - кличка собаки. Китченер назвал своих охотничьих собак Bang
[бэнг], Miss [мисс], Damn [дэмн]. "Муму" - рассказ Тургенева.

Источник:
   1. http://books.google.ru/books?id=7LSeAwAAQBAJ&pg=PT114#v=onepage&q&f=false
   2. И. Тургенев. Муму.

Автор:
Сергей Спешков (Москва)

Вопрос 20:
Героиня романа, который написал Леонид Леонов, после того как всемирная
революция обеспечивала счастье трудящихся, думала, что же дальше, и, как
пишет автор, начинала блуждать в лабиринте ИХ. Назовите ИХ двумя
словами, начинающимися на одну и ту же букву.

Ответ:
Придаточные предложения.

Зачет:
Подчиненные предложения.

Комментарий:
Так Леонов описывает мучения героини при составлении речи, в которой,
очевидно, большинство фраз были до крайности тяжеловесными. Почти как
этот вопрос, равно как и комментарий к нему, равно как и мучения авторов
при составлении оных.

Источник:
Л. Леонов. Собрание сочинений в пяти томах. - М.: Государственное
издательство художественной литературы. - Т. 6 (дополнительный). - С.
212.

Автор:
Дмитрий Великов (Москва)

Вопрос 21:
В этом вопросе слово "ИКС" заменяет другое слово.
   ИКСА известного автора вплоть до 1930-х годов можно было увидеть в
немецких учебниках по биологии, несмотря на наличие более точных работ
Одри и Стаббса, а также фотографий. Назовите ИКСА.

Ответ:
Носорог.

Комментарий:
Речь идет о знаменитой гравюре "Носорог" Альбрехта Дюрера. Для своего
времени она была вполне точна, но всё же много в ней было и выдуманного.
В родной Германии к Дюреру испытывали большое уважение.

Источник:
http://en.wikipedia.org/wiki/D%C3%BCrer's_Rhinoceros

Автор:
Александр Рождествин (Самара)

Вопрос 22:
На плакате, рекламирующем услуги одного специалиста, облака постепенно
принимают привычную правильную форму. О каком специалисте идет речь?

Ответ:
О логопеде.

Комментарий:
Облака в данном случае - то, куда помещаются реплики говорящих.

Источник:
Рекламный плакат по адресу: Москва, ул. Шереметьевская, д. 91, корп. 1.

Автор:
Сергей Спешков (Москва)

Вопрос 23:
Чаплин вспоминал, что его отец был чрезвычайно эгоистичным человеком.
Какое-то время он даже спорил со своей женой, говоря что еще одно ОНО в
семье будет ему мешать. Назовите ЕГО точно.

Ответ:
[Имя] Чарли.

Комментарий:
Чарли Чаплин-младший так вспоминал о своем отце, знаменитом киноактере.
Тот сперва не хотел, чтобы сын был назван в честь него.

Источник:
http://vozduh.afisha.ru/books/charli-chaplin-pitera-akroyda/

Автор:
Александр Рождествин (Самара)

Вопрос 24:
[Ведущему: слово "чьего" в последней фразе вопроса следует прочитать
максимально отчетливо, чтобы команды не услышали "чего".]
   В 1910 году будущий лауреат Нобелевской премии Вернер фон ХЕйдестам
написал книгу "Шведы и их вожди". Людмила БрАуде считает эту книгу
своеобразным продолжением проекта. Чьего?

Ответ:
[Сельмы] ЛАгерлёф.

Комментарий:
Начало этому "проекту" было положено Сельмой Лагерлёф, чья книга о
Нильсе стала своеобразным учебником географии Швеции. Другой нобелиат
написал учебник истории Швеции.

Источник:
   1. http://bookz.ru/authors/sel_ma-lagerlef/udivitel_858/1-udivitel_858.html
   2. http://dic.academic.ru/dic.nsf/enc_colier/5165/

Автор:
Александр Рождествин (Самара)

Вопрос 25:
На рисунке Алексея Булатова человеку, сидящему перед монитором, вручают
ЕЕ со знаком копирайта. Назовите ЕЕ.

Ответ:
Черная метка.

Комментарий:
Карикатура посвящена соблюдению авторских прав. Человек изображен в виде
пирата.

Источник:
http://caricatura.ru/art/bulatov/url/parad/bulatov/3704/

Автор:
Сергей Спешков (Москва)

Вопрос 26:
Дуплет.
   1. В произведении Иосифа Бродского ЕГО настойчивое соло заканчивается
овациями. Назовите ЕГО.
   2. Лирический герой Иосифа Бродского, говоря о НЕЙ, упоминает Музу и
Шиву. Назовите ЕЕ.

Ответ:
   1. Комар.
   2. Муха.

Комментарий:
   1. Зуденье комара, очевидно, заканчивается хлопками.
   2. Крылатая, как муза поэта, и с шестью конечностями.

Источник:
http://lib.ru/BRODSKIJ/brodsky_poetry.txt

Автор:
Сергей Спешков (Москва)

Вопрос 27:
Мёрфи Купер, героиня фильма "ИнтерстЕллар", с детства интересуется
физикой, а вырастая, становится ученым и поступает на работу в НАСА.
Режиссер фильма КрИстофер НОлан вспоминает, что сделал изменение в
изначальном сценарии после некоего события. Ответьте абсолютно точно,
что это было за событие.

Ответ:
Рождение дочери.

Комментарий:
Сперва Мёрфи Купер был мальчиком, однако после рождения дочери Нолан
исправил сценарий, сделав Мёрфи Купер девочкой. Девочка, которая
увлекается физикой и космосом, - довольно необычный персонаж в
голливудских фильмах.

Источник:
http://vozduh.afisha.ru/cinema/kristofer-nolan-stivenu-hokingu-film-ponravilsya/

Автор:
Александр Рождествин (Самара)

Вопрос 28:
[Ведущему: максимально четко произнести название вина айсвАйн.]
   После завершения карьеры известный человек открыл собственную
винодельню. Сколько долларов стоит вино айсвАйн из его коллекции?

Ответ:
99.

Зачет:
99 долларов [США].

Комментарий:
Бывший хоккеист Уэйн Грецки - владелец собственного винного брэнда. С
одной стороны, 99 - игровой номер игрока, с другой стороны, такая цена
соответствует психологическим принципам ценообразования. Айсвайн -
буквально "ледяное вино", для кого-то, возможно, это стало подсказкой.

Источник:
http://www.gretzkyestateswines.com/wines/99-series-1/cabernetfranc-icewine.html

Автор:
Сергей Спешков (Москва)

Вопрос 29:
Психолог ГУстав ГИлберт вспоминает, что к концу ЕГО один из главных
участников всё чаще появлялся без галстука или с расстегнутой пуговицей.
Назовите ЕГО.

Ответ:
Нюрнбергский процесс.

Комментарий:
Речь идет о Риббентропе. Гилберт считал, что на подсознательном уровне
галстук вызывал у Риббентропа ассоциации с петлей виселицы, всё туже
затягивающейся вокруг его шеи.

Источник:
Густав Гилберт. Нюрнбергский дневник.

Автор:
Александр Рождествин (Самара)

Вопрос 30:
В одном литературоведческом комментарии автор вопроса в середине
предложения обнаружил слово "ниже", написанное с большой буквы. Напишите
слово, которое стояло сразу перед ним.

Ответ:
Поручик.

Зачет:
Подпоручик.

Комментарий:
То ли в результате опечатки, то ли по незнанию людей, оцифровывавших
текст, с поручиком Киже произошла еще одна итерация искажения, и он стал
"поручиком Ниже".

Источник:
http://www.james-joyce.ru/ulysses/ulysses-text.htm

Автор:
Сергей Спешков (Москва)

Вопрос 31:
Русские казакИ после НЕЕ продавали лошадей по 15-20 рублей, не зная, что
реальная их стоимость была раз в десять выше. Назовите ЕЕ тремя словами.

Ответ:
Атака легкой кавалерии.

Зачет:
Атака легкой бригады; битва при Балаклаве.

Комментарий:
В этой атаке участвовал цвет британской аристократии. Переловив выживших
лошадей этой, как ее называли в русских войсках, "сумашедшей кавалерии",
казаки, не задумываясь, сбывали их с рук перекупщикам, которые уже
знали, какова истинная цена дорогих кровных рысаков.

Источник:
Е. Тарле. Крымская война.

Автор:
Александр Рождествин (Самара)

Вопрос 32:
Когда этой территории давали название, бОльшая часть американского
континента была еще не освоена европейцами. Ниал Фергюсон считает это
дополнительным основанием для выбора названия. О какой территории идет
речь?

Ответ:
Виргиния.

Зачет:
Вирджиния.

Комментарий:
Не только в честь королевы-девственницы Елизаветы I.

Источник:
Ниал Фергюсон. Империя. Чем современный мир обязан Британии.
http://www.flibusta.net/b/378579/read

Автор:
Сергей Спешков (Москва)

Вопрос 33:
История ИХ появления связана с американскими военными летчиками, которые
не были готовы к мирной жизни, зато были прекрасными пилотами и
разбирались в двигателях. Назовите ИХ двумя словами, начинающимися на
одну ту же букву.

Ответ:
"Ангелы ада".

Комментарий:
По той же легенде, многие из них были из эскадрильи, названной в честь
фильма Говарда Хьюза. Так они объединились и основали байк-клуб, где их
навыки оказались востребованы.

Источник:
http://affa.hells-angels.com/hamc-history/

Автор:
Александр Рождествин (Самара)

Вопрос 34:
Персонаж Уэса Андерсона, говоря о том, как вести себя во время допроса,
упоминает ЕЕ. Чтобы показать ЕЕ на жестовом языке, пальцы одной руки
вначале держат в виде латинской буквы V [вэ], а потом сводят вместе.
Назовите ЕЕ.

Ответ:
Застежка-молния.

Зачет:
Молния, zip, zipper.

Комментарий:
Персонаж говорит, что надо закрыть рот на молнию. В языке жестов пальцы,
которые держат в виде буквы V, означают открытую молнию, которую
движением пальцев второй руки как бы "застегивают".

Источник:
   1. Фильм "Отель "Гранд Будапешт"".
   2. http://www.lifeprint.com/asl101/pages-signs/z/zip.htm

Автор:
Сергей Спешков (Москва)

Вопрос 35:
Выступая с лекцией перед американскими студентами, австриец Виктор
Франкл иронично похвастался, что использует ПЕРВЫЙ без малейших
признаков ВТОРОГО. Назовите ПЕРВЫЙ и ВТОРОЙ в правильном порядке.

Ответ:
Акцент, английский [язык].

Зачет:
Обязательно в правильном порядке!

Комментарий:
Франкл пошутил, что говорит на чистейшем акценте без малейших признаков
английского.

Источник:
http://www.youtube.com/watch?v=fD1512_XJEw&t=33s

Автор:
Александр Рождествин (Самара)

Вопрос 36:
На картине Нормана Роквелла "Русский класс" школьники сидят за партами
перед бюстом Ленина. На этой картине ТАМ на первом месте несколько раз
стоит предпоследняя. Ответьте максимально точно, где - ТАМ?

Ответ:
[В надписи] "Учиться[, учиться и учиться]".

Зачет:
По слову "учиться" или "Yчиться".

Комментарий:
Американский художник не смог до конца справиться с кириллицей и букву
"У" в цитате из Ленина написал как "Y" [уай].

Источник:
   1. http://en.wikipedia.org/wiki/Russian_Schoolroom
   2. http://ru.wikipedia.org/wiki/Файл:Soviet_classroom-rockwell.jpg

Автор:
Сергей Спешков (Москва)

Тур:
Этап 5

Дата:
08-Feb-2015

Редактор:
Михаил Локшин и Дмитрий Петров (Санкт-Петербург)

Инфо:
Редакторы благодарят за тестирование пакета и ценные замечения команды
"Ноев ковчег" (Санкт-Петербург), "40 звездочек" (Санкт-Петербург),
"Суббота 13" (Нью-Йорк), а также Льва Орлова, Ольгу Кузьму, Антона
Чухнова, Кирилла и Маргариту Савицких, Александра Элеазера, Лидию
Иванову, Константина Кнопа, Евгения Поникарова (все - Санкт-Петербург),
Кирилла Чистякова, Максима Дрозда, Михаила Максимова, Вячеслава
Белькова, Александра Иванова (все - Торонто), Марию Гендину (Чикаго),
Мурода Хамроева (Монреаль), Якова Зайдельмана (Переславль-Залесский),
Дмитрия Когана (Фридрихсхафен), Антона Тахтарова (Самара), Евгения
Миротина (Минск), Ивана Ефремова (Ростов-на-Дону), Сергея Лобачёва
(Нижний Новгород), Бориса Белозёрова (Москва), продолжение следует.

Вопрос 1:
[Ведущему: Перед чтением этого вопроса огласите список благодарностей
так, как он приведен в преамбуле к пакету, если вы еще не сделали этого.
После сдачи ответов до или после оглашения правильного ответа прочитайте
полностью комментарий к этому вопросу.]
   В ЮАР полицейские насчитали в спальне одной женщины более десяти
краденых ИХ. Назовите ИХ одним словом.

Ответ:
Овцы.

Зачет:
Овечки.

Комментарий:
Кто-то при бессоннице читает список кораблей, а кто-то - просто считает
овец. Мы список тестеров прочли примерно до середины. Надеемся, он вас
не слишком усыпил.
   Редакторы также благодарят за тестирование пакета и ценные замечания:
клуб знатоков города Винница, Виктора Байрака (Винница - Воронеж), Илону
Косенко (Одесса), Екатерину Дубровскую (Омск), Андрея Гатаулина
(Караганда), Дмитрия Овчарука (Киев), Александра Толесникова
(Тель-Авив), Андрея Погорелова (Тимашевск), Дмитрия Борока (Самара),
Александра Коробейникова, Бориса Гуревича (оба - Саратов), Дмитрия
Соловьева (Нижний Новгород), Фарита Алиби (Астана), Олега Борцова
(Чебоксары), Артема Корсуна (Львов), Анну Овчинникову (Хайфа), Юлию
Устюжанину (Тель-Авив), Вадима Ефимова (Хайфа), Игоря Колмакова
(Тель-Авив).

Источник:
http://www.dailymail.co.uk/news/article-2281768/Cant-pull-wool-eyes-Policeman-finds-men-TEN-sheep-car-animals-allegedly-stolen-farm.html

Автор:
Михаил Локшин, Дмитрий Петров (Санкт-Петербург)

Вопрос 2:
Самый дорогой в истории ОН выпрыгнул с ликом Богоматери и впоследствии
был продан за 28 тысяч долларов. Так пусть и вас ждут приятные сюрпризы!
Назовите ЕГО односложным словом.

Ответ:
Тост.

Комментарий:
На тосте в результате прожарки появился лик Девы Марии.

Источник:
Роберт Хьюз. Рим. История города: его культура, облик, люди. - М.: АСТ:
CORPUS, 2014. - С. 204.

Автор:
Дмитрий Петров (Санкт-Петербург)

Вопрос 3:
Андрей ЩербенОк, говоря о культуре времен застоя, отмечает, что даже
солнечный летний день тогда выглядел тусклым и пыльным. При этом он
упоминает предприятие, находившееся в небольшом украинском городе.
Назовите это предприятие.

Ответ:
Свема.

Зачет:
По слову "Свема" или по словам "Светочувствительные материалы".

Комментарий:
Речь идет о цветном кино, снимавшемся на пленку шосткинского
производственного объединения "Свема" (о чем можно знать из финальных
кадров многих советских фильмов). Но если вы подумали о фотопленке,
ничего страшного.

Источник:
   1. http://magazines.russ.ru/nlo/2013/123/8sh.html
   2. http://ru.wikipedia.org/wiki/Свема

Автор:
Дмитрий Петров (Санкт-Петербург)

Вопрос 4:
Описывая здание заводоуправления, Алексей Иванов замечает, что у входа в
гуще акаций заблудился ОН. В другом произведении ОН осветил путь.
Назовите ЕГО.

Ответ:
Владимир Ильич Ленин.

Зачет:
По фамилии Ленин или Ульянов. Незачет: Памятник Ленину.

Комментарий:
В гимне СССР были слова "И Ленин великий нам путь озарил". Иванов имел в
виду памятник вождю.

Источник:
   1. А. Иванов. Географ глобус пропил.
http://lib.ru/RUFANT/IWANOW_A/geograf_globus_propil.txt
   2. http://ru.wikipedia.org/wiki/Гимн_СССР

Автор:
Михаил Локшин (Санкт-Петербург)

Вопрос 5:
Эмиру Кустурице пришлось сделать 45 дублей эпизода с ЕГО участием.
Режиссер заметил: чтобы снять фильм, важнее быть не талантливым, а
таким, как ОН. В работах Леонида Гайдая и Эндрю Адамсона ОН тоже есть.
Назовите ЕГО одним словом.

Ответ:
Осел.

Зачет:
Ишак, ослик (с отвращением).

Комментарий:
Кустурица в шутку отметил, что важнее быть упрямым, чем талантливым.
Ослик снимался в "Кавказской пленнице". А в "Шреке" Осел - вообще один
из главных персонажей.

Источник:
   1. http://ria.ru/radio_brief/20140806/1019017787.html
   2. http://ru.wikipedia.org/wiki/Кавказская_пленница,_или_Новые_приключения_Шурика
   3. http://ru.wikipedia.org/wiki/Шрек_(мультфильм)

Автор:
Михаил Локшин (Санкт-Петербург)

Вопрос 6:
Подраздел теории графов, изучающий случайные графы, используется в
социальных науках. У встречающихся на практике случайных графов
расстояние между узлами обычно невелико. Назовите ученого,
разработавшего совместно с Альфредом РЕньи теорию случайных графов.

Ответ:
[Пол] Эрдёш.

Комментарий:
"Расстояние" от ученого до Пола Эрдёша через цепочку совместных
публикаций - частный случай расстояния на графе. Свойства этого
расстояния применительно к социальным наукам описываются "теорией
тесного мира", она же "теория шести рукопожатий".

Источник:
http://lenta.ru/articles/2014/02/06/memez/

Автор:
Дмитрий Петров (Санкт-Петербург)

Вопрос 7:
Живущий в приморском городе герой произведения Маркеса каждый вечер
неоднократно на полминуты превращается в полосатого тигра. Причина
метаморфоз - ОН. Если известный список построить по хронологическому
принципу, другой ОН окажется в конце. Назовите ЕГО одним словом.

Ответ:
Маяк.

Комментарий:
Свет маяка проникал сквозь жалюзи. Маркес не уточняет, включался ли
светильник на 30 секунд с интервалами или вращался с заданной
периодичностью. Александрийский маяк - одно из самых поздних по времени
создания из семи чудес света (между 280 и 247 гг. до н.э.).

Источник:
   1. Г. Гарсиа Маркес. Осень патриарха.
http://lib.rin.ru/doc/i/134694p9.html
   2. http://en.wikipedia.org/wiki/Seven_Wonders_of_the_Ancient_World
   3. http://en.wikipedia.org/wiki/Lighthouse_of_Alexandria

Автор:
Михаил Локшин (Санкт-Петербург)

Вопрос 8:
Футуристы известны своими нападками на классиков девятнадцатого века.
Согласно пародийной "Истории советской фантастики", Осип Брик предлагал
фантастам и своим литературным спутникам объединить усилия в борьбе с
искусством прежних лет. Заголовок его статьи об этом на слух мало
отличается от названия, под которым в русском переводе не раз выходило
произведение 1865 года. Воспроизведите любое из этих названий.

Ответ:
"Из Пушкина - Луну!".

Зачет:
"Из пушки на Луну".

Комментарий:
Осип Брик призывал, в том числе, низвергнуть "Солнце русской поэзии".

Источник:
   1. http://www.rulit.net/books/istoriya-sovetskoj-fantastiki-read?151516?5.html
   2. http://en.wikipedia.org/wiki/From_the_Earth_to_the_Moon
   3. http://www.litmir.net/bd/?b=171307

Автор:
Дмитрий Петров (Санкт-Петербург)

Вопрос 9:
Польский исследователь Людвиг Стомма не верит, что сейчас могут быть
какие-то аргументы в ЕГО пользу, кроме одного: ОН дает меньше
возможностей для шантажа в случае преследований. Назовите ЕГО словом
латинского происхождения.

Ответ:
Целибат.

Комментарий:
В странах, где религия тесно связана с политикой, тема преследований за
веру, к сожалению, остается актуальной. Давшего обет безбрачия
священника невозможно шантажировать женой и детьми.

Источник:
   1. Людвиг Стомма. Недооцененные события истории. Книга исторических
заблуждений. - М.: АСТ, 2014. - С. 9.
   2. http://ru.wikipedia.org/wiki/Целибат

Автор:
Дмитрий Петров (Санкт-Петербург)

Вопрос 10:
Мэтт Грейнинг говорит, что в его коллекции пиратской продукции очень
много АЛЬФ с Гомером Симпсоном, возможно, указывая на страну-источник
контрафакта. Скорее всего, в этой коллекции можно встретить и других
персонажей сериала "Симпсоны". Какое слово мы заменили на АЛЬФУ?

Ответ:
Матрешка.

Комментарий:
Семейство Гомера Симпсона - Гомер, Мардж, Лиза, Барт и Мэгги - хорошо
подходит для изображения на наборе матрешек.

Источник:
   1. http://esquire.ru/wil/matt-groening
   2. http://www.artguide.com/posts/319-kartinki-niedieli-12-03-19-03-349
   3. http://azbukivedi.livejournal.com/292426.html
   4. http://joyreactor.cc/post/1695
   5. http://ru.wikipedia.org/wiki/Грейнинг,_Мэтт

Автор:
Дмитрий Петров (Санкт-Петербург)

Вопрос 11:
Томас Джефферсон и Джон Адамс были политическими противниками.
Англоязычная Википедия упоминает интересную особенность картины Джона
ТрАмбулла "Декларация независимости": многим кажется, что в нижней части
полотна Джефферсон [ПРОПУСК] Адамсу. Заполните пропуск тремя словами,
начинающимися на одну и ту же букву.

Ответ:
"... наступает на ногу...".

Зачет:
"... наступил на ногу...".

Источник:
http://en.wikipedia.org/wiki/Declaration_of_Independence_(Trumbull)

Автор:
Дмитрий Петров (Санкт-Петербург)

Вопрос 12:
Один медицинский сайт утверждает, что сам ОН безопасен, но с ЕГО помощью
распространяется пыльца других растений. ЕМУ посвящено произведение 1998
года. Назовите ЕГО двумя словами.

Ответ:
Тополиный пух.

Зачет:
Пух тополя/тополей.

Комментарий:
Опровергая популярное мнение, сайт "Доктор Питер" пишет, что пух - это
семена, а не пыльца. Разнося пыльцу других растений, он вызывает
обострения аллергии. "Тополиный пух" - песня группы "Иванушки
International", вышедшая в 1998 году.

Источник:
   1. http://www.doctorpiter.ru/articles/9069/
   2. http://www.megalyrics.ru/lyric/ivanushki-int/topolinyi-pukh.htm
   3. http://ru.wikipedia.org/wiki/Иванушки_International

Автор:
Михаил Локшин (Санкт-Петербург)

Вопрос 13:
Подопечные Гуса Хиддинка неоднократно вырывали победу за секунды до
финального свистка. При жеребьевке им редко доставались сильные
соперники. Напишите прозвище Хиддинка, двумя буквами отличающееся от
названия известного музыкального произведения.

Ответ:
Везучий голландец.

Комментарий:
У Хиддинка репутация не просто хорошего, но и удачливого тренера.

Источник:
   1. http://ru.wikipedia.org/wiki/Хиддинк,_Гус
   2. http://www.eurosport.ru/football/story_sto945410.shtml
   3. http://ng.sb.by/soyuznoe-veche/article/s-khiddinkom-rossii-stalo-vezti.html

Автор:
Михаил Локшин (Санкт-Петербург)

Вопрос 14:
Один российский тренер сказал о больной спортсменке: "Самочувствие ее
стало хуже. Знаю, что решается вопрос о лечении серьезными
медикаментами, но это уже [ПРОПУСК]". Заполните пропуск словосочетанием
из двух слов, печально известным с середины прошлого века.

Ответ:
"... дело врачей".

Комментарий:
Дело врачей - уголовное дело против группы видных советских
врачей-евреев, которых обвиняли в заговоре и убийстве советских лидеров.
Истоки кампании относятся к 1948 году, окончание - к 1953 году. Медиков
называли "врачами-вредителями". Тренер имел в виду не это, но получилась
двусмысленность.

Источник:
   1. http://www.itar-tass.com/sport/978633
   2. http://ru.wikipedia.org/wiki/Дело_врачей

Автор:
Михаил Локшин (Санкт-Петербург)

Вопрос 15:
[Ведущему: не сообщать о том, что слово "Лицо" написано с заглавной
буквы.]
   Пушкин, заметив сидевшего между Булгариным и Гречем лицеиста
Семенова, вспомнил аналогичную ситуацию и упомянул известное Лицо и
топоним. Какой именно топоним?

Ответ:
Голгофа.

Комментарий:
Неприязненные отношения Пушкина и Булгарина общеизвестны. Обращаясь к
Семенову, поэт язвительно поддел его соседей: "Ты, брат Семенов, словно
Христос на горе Голгофе!". Так он намекал на то, что Спаситель был
распят на кресте между двумя разбойниками. Иисус Христос - Второе Лицо
Святой Троицы.

Источник:
   1. http://www.spbvedomosti.ru/print.htm?id=10298989@SV_Articles
   2. http://ru.wikipedia.org/wiki/Троица

Автор:
Михаил Локшин (Санкт-Петербург)

Вопрос 16:
Внимание, в вопросе есть замены.
   В Ветхом завете говорится о голодных годах, в которые - цитата - "ни
ПИТЬ, ни ЗАКУСЫВАТЬ не будут". При игре в брэйн-ринг необходимо
ЗАКУСЫВАТЬ, а в некоторых случаях эмоциональные знатоки могут ПИТЬ.
Какие глаголы мы заменили на "ПИТЬ" и "ЗАКУСЫВАТЬ"?

Ответ:
Орать, жать.

Зачет:
В любом порядке.

Комментарий:
В первом случае "орать" - пахать, "жать" - пожинать. Во втором "орать" -
кричать, "жать" - нажимать на кнопку.

Источник:
   1. http://days.pravoslavie.ru/Bible/B_byt45.htm
   2. http://dic.academic.ru/dic.nsf/enc2p/291570/
   3. ЛОАВ.

Автор:
Михаил Локшин (Санкт-Петербург)

Вопрос 17:
(pic: 20140521.jpg)
   Умер осенью. Жил в семнадцатом веке. Как его зовут?

Ответ:
Мацуо Басё.

Зачет:
По слову Басё; по имени Мацуо, Мунэфуса или Дзинситиро.

Комментарий:
   Весь этот вопрос
   Напоминает хокку.
   Судите сами:
   Умер осенью.
   Жил в семнадцатом веке.
   Как его зовут?
   В "правильном" хокку
   Нужно указание
   На время года.

Источник:
   1. http://ru.wikipedia.org/wiki/Мацуо_Басё
   2. http://www.world-art.ru/people.php?id=10799
   3. http://ru.wikipedia.org/wiki/Хайку

Автор:
Михаил Локшин (Санкт-Петербург)

Вопрос 18:
[Ведущему: кавычки не озвучивать.]
   Для взятия вопроса не обязательно быть поклонником ЭТОГО.
   Действие ЭТОГО, вышедшего в 1998 году, происходит в будущем. Люди
колонизировали Солнечную систему, в том числе - небесное тело, в котором
можно найти "ЭТО". Назовите небесное тело.

Ответ:
Ганимед.

Комментарий:
В слове "Ганимед" можно найти слово "аниме". В фантастическом
аниме-сериале "Cowboy Bebop" Ганимед и некоторые другие спутники Юпитера
колонизированы.

Источник:
   1. http://www.imdb.com/title/tt0213338/
   2. http://en.wikipedia.org/wiki/Cowboy_Bebop

Автор:
Дмитрий Петров (Санкт-Петербург)

Вопрос 19:
Сергей ДорогАвцев заметил, что торги на бирже в некий момент проходили
под знаком ЕГО: на фоне новостей из США дешевела нефть, снижались акции.
Назовите ЕГО несклоняемым словом.

Ответ:
Гризли.

Комментарий:
Трейдеров, которые ожидают понижения цен, называют медведями. Медведи
придавливают соперника лапами вниз. Гризли - типичный для Америки
медведь.

Источник:
   1. http://www.finam.ru/analysis/marketnews3DDE5/default.asp
   2. http://ru.wikipedia.org/wiki/Гризли
   3. http://ru.wikipedia.org/wiki/Трейдер

Автор:
Михаил Локшин (Санкт-Петербург)

Вопрос 20:
[Ведущему: не пропустить преамбулу к дуплету!]
   Дуплет. Два вопроса по 30 секунд обсуждения каждый, ответы сдаются на
одном бланке. Чтобы получить очко, вам нужно правильно ответить на оба
вопроса.
   1. Скульптура ЕГО на крыше в одном украинском городе кому-то
показалась неуместной. Назовите ЕГО.
   2. По сообщению журнала "Популярная механика", одной австралийской
лошади ЕГО отчасти заменил 3D-принтер. Назовите ЕГО.

Ответ:
   1. Скрипач.
   2. Кузнец.

Комментарий:
Как известно из фильмов "Кин-дза-дза!" и "Формула любви", "скрипач не
нужен" и "нам кузнец не нужен". Австралийская скаковая лошадь получила
титановые подковы, индивидуально изготовленные на 3D-принтере на основе
отсканированных моделей копыт.

Источник:
   1. http://ru.wikipedia.org/wiki/Скрипач_на_крыше_(скульптура)
   2. http://www.popmech.ru/article/14055-a-zachem-nam-kuznets/
   3. http://ru.wikiquote.org/wiki/Кин-дза-дза!
   4. http://ru.wikiquote.org/wiki/Формула_любви

Автор:
Дмитрий Петров (Санкт-Петербург)

Вопрос 21:
[Ведущему: отточия в цитате игнорировать.]
   Внимание, в вопросе слова "ОДИН" и "ДРУГОЙ" - замены.
   Цитата из интервью: "Мне всегда был симпатичен ОДИН. ДРУГОЙ...
замкнутый, слишком сосредоточенный на своей миссии. У него короткая
борода... ОДИН - ... жизнелюб, который запросто может выпить с друзьями,
любит веселье, хорошие шутки" (конец цитаты). Так британский актер
высказался о своей роли. Ответьте в правильном порядке: кто такой ОДИН и
кто - ДРУГОЙ?

Ответ:
Гэндальф Серый, Гэндальф Белый.

Зачет:
В правильном порядке.

Комментарий:
ОДИН и ДРУГОЙ - это Гэндальф Серый и Гэндальф Белый соответственно
("Один серый, другой - белый"; кстати, Толкин представлял Гэндальфа
похожим на Одина). Интервью дал Иэн Маккеллен, сыгравший Гэндальфа в
трилогиях "Властелин Колец" и "Хоббит".

Источник:
   1. "Панорама ТВ", 2013, N 5. - С. 84.
   2. http://ru.wikipedia.org/wiki/Гэндальф

Автор:
Михаил Локшин (Санкт-Петербург)

Вопрос 22:
На гобелене из Байё можно увидеть пронзенного стрелой английского короля
ГАрольда. Эндрю УИттакер сравнил это изображение с кадрами кинопленки,
снятой оператором-любителем... В каком городе?

Ответ:
В Далласе.

Комментарий:
Абрахам ЗапрУдер запечатлел убийство Джона Кеннеди. Гобелен из Байё -
длинный, узкий и по композиции напоминает кинопленку.

Источник:
Э. Уиттакер. Великобритания. - М.: РИПОЛ классик, 2013. - С. 29.

Автор:
Дмитрий Петров (Санкт-Петербург)

Вопрос 23:
Фрэнк МаклИнн назвал вождя чЕроки, настаивавшего на переговорах с
губернатором Южной Каролины, единственным ПЕРВЫМ в племени ВТОРЫХ.
Назовите ПЕРВЫХ и ВТОРЫХ.

Ответ:
Голуби, ястребы.

Комментарий:
Оппоненты вождя хотели войны. Применение к миролюбивым и воинственным
индейцам терминов "голуби" и "ястребы" выглядит довольно забавно.

Источник:
Ф. Маклинн. 1759. Год завоевания Британией мирового господства. - М.:
АСТ: АСТ МОСКВА, 2009. - С. 33.

Автор:
Дмитрий Петров (Санкт-Петербург)

Вопрос 24:
Линейные крейсера класса "Конго" сравнивали с НЕЙ за сужающиеся к верху
высокие надстройки. ОНА, установленная около двух месяцев назад в
приграничной зоне на территории Южной Кореи, расценивается Севером как
орудие психологической войны. Назовите ЕЕ двумя словами.

Ответ:
Рождественская елка.

Зачет:
Новогодняя елка; рождественская ель; новогодняя ель.

Комментарий:
Высокие надстройки со множеством прожекторов ночью светились, как
рождественская ель. Светящаяся рождественская елка в южнокорейской
приграничной зоне воспринимается в КНДР как средство психологической
войны: у нас тут праздник и веселье, а у вас - нет.

Источник:
   1. http://en.wikipedia.org/wiki/Pagoda_mast
   2. http://english.yonhapnews.co.kr/national/2014/12/02/94/0301000000AEN20141202003800315F.html

Автор:
Дмитрий Петров (Санкт-Петербург)

Вопрос 25:
Короткое знаменитое высказывание ОН связал с поэзией. Ему вспомнилась
строка Маяковского о незнаемом. Действительно, что-то подобное
предстояло тогда и ему самому. Разъяснения какого известного человека мы
вам пересказали?

Ответ:
[Юрия Алексеевича] Гагарина.

Комментарий:
"Я думаю, это относилось к термину "езда в незнаемое". Ведь так говорил
Владимир Маяковский о поэзии. А что может быть поэтичнее космоса?". Так
Гагарин объяснил свое знаменитое "Поехали!". Первые буквы предложений
вопроса составляют слово "кедр" - позывной Гагарина.

Источник:
   1. http://www.piter.fm/share/7873/20140311/fromtime:08:48:22
   2. http://ru.wikipedia.org/wiki/Гагарин,_Юрий_Алексеевич
   3. http://az.lib.ru/m/majakowskij_w_w/text_0890.shtml

Автор:
Михаил Локшин (Санкт-Петербург)

Вопрос 26:
Один изобретатель придумал чудо-машину, помогающую СДЕЛАТЬ ЭТО за три
дня. Чтобы СДЕЛАТЬ ЭТО в три этапа, пригодится название денежных единиц
двух азиатских государств. Широко известен случай, когда ЭТО БЫЛО
СДЕЛАНО... Ответьте точно: где именно?

Ответ:
В Кане [Галилейской].

Зачет:
По упоминанию Каны ("на брачном пире в Кане" и т.п.).

Комментарий:
СДЕЛАТЬ ЭТО - превратить воду в вино. По словам изобретателя Филиппа
Джеймса, машина использует водопроводную воду и сокращает процесс
виноделия до трех дней. Слово "вода" превращается в слово "вино" в три
этапа: вода - вона - вина - вино. И в Республике Корея, и в КНДР
используются воны. В Кане Галилейской Иисус Христос совершил чудо -
превратил воду в вино.

Источник:
   1. http://www.gazeta.ru/tech/news/2014/03/07/n_5997049.shtml
   2. http://www.iqfun.ru/articles/muha-slon.shtml
   3. http://ru.wikipedia.org/wiki/Северокорейская_вона
   4. http://ru.wikipedia.org/wiki/Южнокорейская_вона
   5. http://ru.wikipedia.org/wiki/Кана_Галилейская

Автор:
Михаил Локшин (Санкт-Петербург)

Вопрос 27:
Литературовед Лоурэнс Дэнсон пишет, что к середине девятнадцатого века
ОН стал более женственным. ОН перестал быть деятельным мстителем, стал
более пассивным, задумчивым, сомневающимся. Назовите его имя.

Ответ:
Гамлет.

Комментарий:
По мнению Дэнсона, в интерпретации современников образ становился всё
более "женственным". Его даже стали играть женщины, в том числе - Сара
Бернар.

Источник:
http://muse.jhu.edu/journals/shq/summary/v059/59.1danson.html

Автор:
Дмитрий Петров (Санкт-Петербург)

Вопрос 28:
Некоторые считают, что колесно-гусеничный танк А-20 был разработан
специально для ИКСОВ. В онлайн-игре "Мир танков" ИКС можно получить,
например, за стрельбу по союзникам. Какое слово мы заменили на ИКС?

Ответ:
Автобан.

Комментарий:
Некоторые вслед за Виктором Суворовым считают, что танк разработан
специально для немецких скоростных трасс и буква "A" значит
"автострадный". Автобан - это еще и автоматический бан пользователя.

Источник:
   1. http://lurkmore.to/Автострадный_танк
   2. http://ru.wikipedia.org/wiki/А-20_(танк)
   3. http://forum.worldoftanks.ru/index.php?/topic/97748-автобан

Автор:
Дмитрий Петров (Санкт-Петербург)

Вопрос 29:
Дуплет. Два вопроса по 30 секунд обсуждения каждый, ответы сдаются на
одном бланке.
   1. Угандиец Джон АкИи-БуА удостоился особых почестей, но позже был
вынужден бежать из страны. Впоследствии АкИи-БуА был принят на работу в
известную компанию. Какую?
   2. В некоторых речевых информаторах серии РИ [эр-и] сообщения
передавались женским голосом. За это система получила от летчиков
уменьшительное прозвище. Какое?

Ответ:
   1. "Puma".
   2. "Рита".

Комментарий:
Джон Акии-Буа, олимпийский чемпион в беге с препятствиями, был особо
отмечен Иди Амином, но после смены власти ему пришлось покинуть страну.
Слово "пума", записанное латинскими буквами, можно при желании прочитать
как "Рита". "Рита" - это уменьшительная форма имени "Маргарита".

Источник:
   1. http://en.wikipedia.org/wiki/John_Akii-Bua
   2. http://ru.wikipedia.org/wiki/Речевой_информатор

Автор:
Дмитрий Петров (Санкт-Петербург)

Вопрос 30:
[Ведущему: кавычки не озвучивать.]
   Отца "отца" отца звали Эдвард. "Отец" отца в 1922 году принял
католичество. Назовите фамилию Эдварда.

Ответ:
Честертон.

Комментарий:
Литературный отец католического священника отца Брауна Гилберт Кит
Честертон тоже стал католиком.

Источник:
http://www.catholicauthors.com/chesterton.html

Автор:
Михаил Локшин (Санкт-Петербург)

Вопрос 31:
В стихотворении Владимира САлимона церквушка на холме, уцелевшая при
пожаре, кажется, может потерять равновесие и упасть от малейшего
дуновения ветра. Поэт сравнивает ее с героиней произведения 1905 года.
Какого?

Ответ:
"Девочка на шаре".

Комментарий:
   Совсем как девочка на шаре,
   стоит церквушка на холме,
   что уцелела при пожаре
   Москвы...

Источник:
   1. http://magazines.russ.ru/arion/2014/2/1s.html
   2. http://ru.wikipedia.org/wiki/Девочка_на_шаре

Автор:
Дмитрий Петров (Санкт-Петербург)

Вопрос 32:
После 2011 года оставшиеся ОНИ попали в музеи. Один американец в связи с
этим с грустью вспоминает запоздалый дребезг оконных стекол. Назовите ИХ
словом с удвоенной согласной.

Ответ:
Шаттлы.

Комментарий:
В 2011 году программа полетов шаттлов была свернута. Корабли "Атлантис",
"Дискавери", "Индэвор" и "Колумбия" были переданы различным музеям США.
Американец с ностальгией вспоминает, как когда-то видел пролетающий на
сверхзвуковой скорости шаттл. Больше над его городом шаттлы не летают.

Источник:
   1. http://ru.wikipedia.org/wiki/Спейс_шаттл
   2. http://io9.com/that-was-the-shuttle-for-me-i-remember-one-day-when-t-1626890085

Автор:
Дмитрий Петров (Санкт-Петербург)

Вопрос 33:
Половина ответа присутствует в вопросе.
   По одной из версий, ЕГО название происходит от нижненемецкого "de
engil" [дэ Энгиль]: считается, что о его целебных свойствах людям
поведал ангел. О НЕМ напоминает фамилия человека, умершего на чужбине.
Ответьте одним словом: что принято оставлять на могиле этого человека?

Ответ:
Пуанты.

Комментарий:
ОН - дягиль. По одной из версий, слово происходит от нижненемецкого "de
engil", "ангел". Человек - Сергей Дягилев, на могиле которого в Венеции
хореографы и танцовщики по традиции оставляют балетные туфли. В этом
вопросе тоже есть пуант (как заключительная часть шутки, загадки или
вопроса "Что? Где? Когда?").

Источник:
   1. http://en.wikipedia.org/wiki/Angelica_archangelica
   2. http://ru.wikipedia.org/wiki/Дягиль_лекарственный
   3. http://ru.wikipedia.org/wiki/Дягилев,_Сергей_Павлович
   4. http://ru.wikipedia.org/wiki/Пуанты
   5. http://dic.academic.ru/dic.nsf/dic_synonims/144775/
   6. http://chgk.wikia.com/wiki/Сленг

Автор:
Дмитрий Петров (Санкт-Петербург)

Вопрос 34:
Авторы называли ЕЕ символом движения человека к Будущему - медленного,
изнурительного, но неуклонного. Назовите объект Всемирного наследия
ЮНЕСКО, который с НЕЙ связан.

Ответ:
Фудзияма.

Зачет:
Фудзи, Фудзисан.

Комментарий:
Так Стругацкие разъясняют смысл взятого в качестве эпиграфа хокку
Кобаяси Иссы "Тихо, тихо ползи, улитка, по склону Фудзи, вверх, до самых
высот".

Источник:
   1. Б.Н. Стругацкий. Комментарии к пройденному.
http://www.litmir.me/br/?b=84548&p=17
   2. http://ru.wikipedia.org/wiki/Фудзияма

Автор:
Михаил Локшин (Санкт-Петербург)

Вопрос 35:
Один из ЕЕ вариантов, появившийся в 1931 году, англоязычная Википедия
сравнивает с изображением электрической цепи. На постере Дэвида Бута
линии на НЕЙ получаются из тюбиков с красками. Назовите ЕЕ тремя
словами.

Ответ:
Карта лондонского метрополитена.

Зачет:
Карта/схема лондонского метро[политена], карта/схема метро[политена]
Лондона.

Комментарий:
Известное название лондонского метро - "tube" (т.е. "труба") - созвучно
слову "тюбик" (и омонимично английскому слову "tube" с соответствующим
значением). До 1931 года карты метро были привязаны к географическим
картам. Сотрудник лондонской подземки Гарри Бек сделал первую карту
метро в виде топологической диаграммы, что сделало ее более читаемой.

Источник:
http://en.wikipedia.org/wiki/Tube_map

Автор:
Дмитрий Петров (Санкт-Петербург)

Вопрос 36:
Мостранссоюз решил принять ряд мер, чтобы распрощаться с нелегальным
частным извозом. Рассказывая об этом, один сайт процитировал припев
зарубежной песни, чуть изменив второе слово из трех. Напишите то, что
получилось в результате.

Ответ:
"Чао, "бомбила", сорри".

Зачет:
С произвольной пунктуацией.

Комментарий:
Частных таксистов, особенно нелегальных, называют бомбилами. В оригинале
- "Чао, бамбино, сорри". Слово "распрощаться" - подсказка.

Источник:
   1. http://www.solidarnost.org/thems/obshestvo/obshestvo_8260.html
   2. http://megalyrics.ru/lyric/mireille-mathieu/ciao-bambino-sorry.htm

Автор:
Михаил Локшин (Санкт-Петербург)

Тур:
Этап 6

Дата:
01-Mar-2015

Редактор:
Сергей Лобачёв и Ярослав Косарев (Нижний Новгород)

Инфо:
Редакторы благодарят за тестирование вопросов и помощь в подготовке
пакета: Якова Зайдельмана (Переславль-Залесский), Антона Тахтарова
(Самара - Рамат-Ган), Вадима Молдавского (Лондон), Бориса Белозёрова,
Александра Карчевского (оба - Москва), Ивана Ефремова (Ростов-на-Дону),
Николая Слюняева, Алексея Сергеева, Илью Редькина, Михаила Царёва,
Алексея Новолокина, Антона Седова, Егора Господчикова, Марию Лазареву
(все - Нижний Новгород).

Вопрос 1:
Во время эвакуации жителей многоквартирных домов работникам газовых
компаний запрещается пользоваться ИКСАМИ. Один дизайнерский ИКС
стилизован под клавишу "Enter". Назовите ИКС.

Ответ:
Дверной звонок.

Зачет:
Звонок; Входной звонок; Электрический звонок.

Комментарий:
Электрический звонок может спровоцировать взрыв газа.

Источник:
   1. https://www.nipsco.com/stay-safe/first-responders/preventing-natural-gas-ignition
   2. http://www.ubergizmo.com/2008/09/enterbell-concept/

Автор:
Сергей Лобачёв (Нижний Новгород)

Вопрос 2:
По словам ландшафтного архитектора Пита Ди, настоящий ОН доберется и до
вершины Эвереста, если там будет установлен флажок. Назовите ЕГО.

Ответ:
Гольфист.

Зачет:
Игрок в гольф.

Комментарий:
Как известно, лунки для гольфа оборудуются флажками. Ландшафтный
дизайнер Питер Ди, упомянутый в вопросе, занимался также и
проектированием полей для гольфа.

Источник:
http://en.wikipedia.org/wiki/Portal:Golf/Quotes_archive

Автор:
Сергей Лобачёв (Нижний Новгород)

Вопрос 3:
Внимание, цитата! "Я странник. Я пришел с миром. Отведите меня к своему
вождю, и вы будете награждены навеки". Конец цитаты. Специально для
Джона перевод этой фразы был подготовлен сразу на несколько языков.
Назовите фамилию Джона.

Ответ:
Гленн.

Комментарий:
Место посадки американского астронавта Джона Гленна не могло быть точно
рассчитано.

Источник:
   1. http://www.futilitycloset.com/2014/05/06/to-whom-it-may-concern-3/
   2. http://ru.wikipedia.org/wiki/Гленн,_Джон

Автор:
Сергей Лобачёв (Нижний Новгород)

Вопрос 4:
В сериале "Куприн" - экранизации нескольких произведений писателя -
неизменным героем всех серий является сам Куприн. Говоря о его роли в
сериале, автор одной рецензии сравнивает телезрителей с ПЕРВЫМ, а
Куприна со ВТОРЫМ. Кого мы заменили на ПЕРВОГО и ВТОРОГО?

Ответ:
Данте, Вергилий.

Комментарий:
Куприн словно проводит зрителя по своему творчеству, как Вергилий
проводил Данте по чистилищу и аду.

Источник:
http://www.kinopoisk.ru/user/497065/comment/2118167/

Автор:
Ярослав Косарев (Нижний Новгород)

Вопрос 5:
Чарльз Бэббидж СДЕЛАТЬ ЭТОГО не сумел, но отрицательный результат
вдохновил его на создание вычислительной машины. Назовите уроженца
городка Дрё, который был одним из немногих, сумевших СДЕЛАТЬ ЭТО.

Ответ:
[Франсуа-Андре Даникан] Филидор.

Комментарий:
Чарльз Бэббидж не сумел победить шахматный автомат "Турок". А вот
известному шахматисту того времени Франсуа-Андре Филидору это удалось.

Источник:
   1. http://science.slashdot.org/story/02/04/21/1719215/1770-mechanical-chess-player-inspired-babbage
   2. http://en.wikipedia.org/wiki/The_Turk

Автор:
Сергей Лобачёв (Нижний Новгород)

Вопрос 6:
Автогонщик Даниил Квят сказал, что с приходом в "Формулу-1" он в
некоторых случаях стал использовать "более быструю" АЛЬФУ. Назовите
АЛЬФУ одним словом.

Ответ:
Подпись.

Зачет:
Роспись.

Комментарий:
После перехода в "Формулу-1" Квяту пришлось чаще давать автографы, при
этом на оригинальную роспись уходит больше времени.

Источник:
http://www.championat.com/auto/article-196537-kvjat---o-toro-rosso-avtografakh-i-futbole.html

Автор:
Ярослав Косарев (Нижний Новгород)

Вопрос 7:
В своем романе "Война миров" Герберт Уэллс призывает не судить
инопланетян строго и напоминает читателю о судьбе ИКСОВ. Назовите
соотечественника Уэллса, носившего прозвище ИКС.

Ответ:
[Чарльз Лютвидж] Доджсон.

Зачет:
[Льюис] Кэрролл.

Комментарий:
Писатель призывает вспомнить о судьбе дронтов, полностью истребленных
человеком. Широко распространено мнение, что Доджсон заикался и часто
произносил свою фамилию как "До-До-Доджсон", за что и получил прозвище
"Додо".

Источник:
   1. http://lib.ru/INOFANT/UELS/warworld.txt
   2. http://en.wikipedia.org/wiki/Dodo_(nickname)

Автор:
Сергей Лобачёв (Нижний Новгород)

Вопрос 8:
В своем последнем письме Циолковскому Беляев извиняется за то, что
ДЕЛАЕТ ЭТО. Вопреки распространенной версии, ДЕЛАТЬ ЭТО в космосе было
опасно. Какие два слова мы заменили словами "ДЕЛАТЬ ЭТО"?

Ответ:
Писать карандашом.

Комментарий:
Беляев был уже тяжело болен и буквально прикован к постели. Поэтому он
не мог пользоваться чернилами и ему приходилось писать карандашом.
Вопреки известной городской легенде, советские космонавты не
использовали карандаши на борту орбитальных станций, так как плавающие
по космическому кораблю частички графита представляют опасность для
экипажа и техники.

Источник:
   1. http://magazines.russ.ru/neva/2005/4/kudr25.html
   2. http://inosmi.ru/world/20121118/202319832.html

Автор:
Сергей Лобачёв (Нижний Новгород)

Вопрос 9:
Герой одного фантастического рассказа совершает удивительное открытие во
время экспедиции по затерянным районам Центральной Америки. Сетуя на
отсутствие доказательств, которые он мог бы предъявить научному миру, он
упоминает ИКСА. Назовите ИКСА словом с двумя греческими корнями.

Ответ:
Птеродактиль.

Комментарий:
Рассказ содержит отсылку к роману Конан Дойля "Затерянный мир", героям
которого удалось привезти с собой в Лондон птеродактиля.

Источник:
Хюберт Лампо. Рождение бога. http://www.flibusta.net/b/216698/read

Автор:
Сергей Лобачёв (Нижний Новгород)

Вопрос 10:
Рассказывая о Стивене Хокинге, один журнал пишет, что в университетские
годы он занимался спортом, однако был... Ответьте одним словом, кем
именно.

Ответ:
Рулевым.

Комментарий:
Стивен обучался в Британии, где академическая гребля и по сей день
является весьма популярным видом спорта.

Источник:
"Машины и механизмы", 2014, N 10.

Автор:
Сергей Лобачёв (Нижний Новгород)

Вопрос 11:
Во время певческих выступлений императора Нерона никому не позволялось
выходить из театра. Поэтому иногда во время таких выступлений некоторые
люди ДЕЛАЛИ ЭТО. Тауэрские вороны иногда ДЕЛАЮТ ЭТО, злостно подшучивая
над своим смотрителем. Что мы заменили словами "ДЕЛАТЬ ЭТО"?

Ответ:
Притворяться мертвым.

Зачет:
Притворяться мертвыми; Симулировать смерть.

Комментарий:
Люди притворялись мертвыми, чтобы их выносили из театра. История
умалчивает относительно дальнейшей судьбы таких хитрецов. Тауэрские
вороны же подобным образом играют на нервах своего смотрителя.

Источник:
   1. http://www.bards.ru/press/press_show.php?id=238
   2. "Англия в общем и в частности", 2-я серия, 15-я минута.

Автор:
Ярослав Косарев (Нижний Новгород)

Вопрос 12:
По словам Юрия Лопухина, репутация российских специалистов настолько
высока, что сотрудники лаборатории при этом учреждении время от времени
посещают Ханой. Назовите это учреждение двумя словами.

Ответ:
Мавзолей Ленина.

Комментарий:
Упомянутые сотрудники периодически приводят в порядок забальзамированное
тело Хо Ши Мина.

Источник:
http://www.mk.ru/science/article/2010/05/18/491259-ho-shi-min-umiral-pod-obektivom-kinokameryi-foto.html

Автор:
Сергей Лобачёв (Нижний Новгород)

Вопрос 13:
В фантастическом романе "Свидание с Рамой" говорится, что к
определенному моменту римский и греческий ИКСЫ закончились. Какое слово
мы заменили на ИКС?

Ответ:
Пантеон.

Комментарий:
Персонажи греческой и римской мифологии закончились, и называть
космические объекты пришлось в честь богов других культур. Таким образом
очередной космический объект получил имя "Рама".

Источник:
Артур Кларк. Свидание с Рамой. http://lib.ru/KLARK/rama1.txt

Автор:
Сергей Лобачёв (Нижний Новгород)

Вопрос 14:
Адвокат "Ювентуса", оспаривая дисквалификацию Златана Ибрагимовича,
утверждал, что того неоднократно провоцировали соперники. При этом
адвокат воспользовался услугами человека, который умеет ДЕЛАТЬ ЭТО.
Согласно одному сайту, учиться ДЕЛАТЬ ЭТО "можно всегда и везде: в кафе,
в метро, на улице, на работе, на отдыхе". Что мы заменили словами
"ДЕЛАТЬ ЭТО"?

Ответ:
Читать по губам.

Комментарий:
Человек, умеющий читать по губам, при просмотре записи матча смог
засвидетельствовать, какие именно фразы говорили соперники Златану.
Согласно упомянутому сайту, чтобы научиться читать по губам, нужно
смотреть на людей вокруг и пытаться понять то, что они говорят.

Источник:
   1. Златан Ибрагимович. Я - Златан.
http://www.sports.ru/tribuna/blogs/legaserie/556064.html
   2. http://www.kak-bog.ru/kak-nauchitsya-chitat-po-gubam

Автор:
Ярослав Косарев (Нижний Новгород)

Вопрос 15:
По одной из версий, ПЕРВАЯ символизирует две скрещенные руки, а ВТОРАЯ
была выбрана за сходство с ладонью. Изобразите ПЕРВУЮ и ВТОРУЮ в
правильном порядке.

Ответ:
XV.

Зачет:
X, V.

Комментарий:
Речь идет о римских цифрах.

Источник:
Айзек Азимов. О времени, пространстве и других вещах.
http://www.flibusta.net/b/358185/read

Автор:
Сергей Лобачёв (Нижний Новгород)

Вопрос 16:
Фрэнк Стэннард считал, что объекты, находящиеся в двух параллельных
вселенных, могут существовать не сталкиваясь между собой. При этом он
упоминает две пары соперников, играющих... Во что?

Ответ:
В шашки.

Комментарий:
На черных и белых полях одной доски могут проводиться две партии
одновременно.

Источник:
Джеймс Линдон. Как окрестить червя.
http://www.flibusta.net/b/364464/read

Автор:
Сергей Лобачёв (Нижний Новгород)

Вопрос 17:
В фильме "Бесы" есть сцена нелегальной сходки. Одна опасливая героиня
перед важным обсуждением просит ИКСА приступить к делу. 23 марта 2003
года "ИКСА" обошло "Чикаго". Какое слово мы заменили на ИКС?

Ответ:
Пианист.

Комментарий:
Героиня просила начать играть, чтобы обсуждение революционеров никто не
подслушал. В 2003 году "Оскар" за лучший фильм получил мюзикл "Чикаго",
а "Пианист" Романа Полански был среди номинантов.

Источник:
   1. Сериал "Бесы", реж. В. Хотиненко.
   2. http://ru.wikipedia.org/wiki/Оскар_(кинопремия,_2003)

Автор:
Ярослав Косарев (Нижний Новгород)

Вопрос 18:
В средние века в Англии считали, что того, кто весел и крепок духом,
минует ужасная участь. Поэтому ОН, по мнению Игоря Шайтанова, - это не
метафора, а довольно частое явление в те времена. Назовите ЕГО.

Ответ:
Пир во время чумы.

Комментарий:
Согласно распространенным в те времена поверьям, спасти от чумы могут не
только снадобья, но и хорошее морально-психологическое состояние,
которому способствовали различные увеселения.

Источник:
И. Шайтанов. Шекспир. http://www.flibusta.net/b/366509/read

Автор:
Ярослав Косарев (Нижний Новгород)

Вопрос 19:
Внимание, в вопросе есть замена.
   "Географические наименования в честь ученых, государственных и
общественных деятелей, военачальников и воинов, корифеев искусства и
мастеров литературы - это, образно говоря, вторая серия". Какие три
слова мы заменили одним в этой цитате из книги Эдуарда Вартаньяна?

Ответ:
Жизнь замечательных людей.

Комментарий:
Заменено было слово "серия". "Жизнь замечательных людей" - серия
биографических книг о жизни замечательных людей.

Источник:
Э. Вартаньян. История с географией, или Жизнь и приключения
географических названий. http://www.flibusta.net/b/106518/read

Автор:
Ярослав Косарев (Нижний Новгород)

Вопрос 20:
Одно из самых своих известных произведений в жанре поэзии ЭТОТ ЧЕЛОВЕК
написал под псевдонимом C.3.3 [си три три]. Назовите ЭТОГО ЧЕЛОВЕКА.

Ответ:
[Оскар] Уайлд.

Комментарий:
Речь идет о "Балладе Редингской тюрьмы", написанной Оскаром Уайлдом.
Уайлд отбывал в этой тюрьме двухлетний срок, и C.3.3 - это его номер
заключенного, который означал третью камеру третьей площадки галереи C.

Источник:
http://ru.wikipedia.org/wiki/Баллада_Редингской_тюрьмы

Автор:
Ярослав Косарев (Нижний Новгород)

Вопрос 21:
Герой романа Нила Стивенсона упоминает барабанный бой и сравнивает ЕЕ с
исполинской сороконожкой. Назовите ЕЕ одним словом.

Ответ:
Галера.

Зачет:
Триера.

Комментарий:
Барабаны задавали ритм гребцам, налегавшим на весла. Весельная галера
имеет сходство с сороконожкой.

Источник:
Нил Стивенсон. Король бродяг. http://www.flibusta.net/b/365885/read

Автор:
Сергей Лобачёв (Нижний Новгород)

Вопрос 22:
Компания "Caterpillar" [катерпИллер] в качестве демонстрации точности
своей техники провела показ игры в гигантскую АЛЬФУ. Назовите АЛЬФУ
словом, пришедшим из языка суахили.

Ответ:
Дженга.

Комментарий:
"Caterpillar" [катерпИллер] - компания, являющаяся одним из мировых
лидеров по производству спецтехники. В рекламном ролике показано, как
погрузчик и экскаватор играют в дженгу, двигая огромные деревянные
блоки.

Источник:
   1. http://metkere.com/2014/04/catjenga.html
   2. http://en.wikipedia.org/wiki/Jenga

Автор:
Сергей Лобачёв (Нижний Новгород)

Вопрос 23:
Олдос Хаксли скончался 22 ноября 1963 года. Рассказывая об этом, Дмитрий
Шушарин упоминает другого деятеля культуры. Назовите его фамилию.

Ответ:
Прокофьев.

Комментарий:
Смерть Олдоса Хаксли затмило более громкое событие - убийство Джона
Кеннеди. Схожая ситуация имела место в 1953 году, когда в один день
скончались Иосиф Сталин и Сергей Прокофьев.

Источник:
   1. http://en.wikipedia.org/wiki/Aldous_Huxley
   2. http://en.wikipedia.org/wiki/John_F._Kennedy
   3. https://www.facebook.com/dshush/posts/715909355103949/

Автор:
Сергей Лобачёв (Нижний Новгород)

Вопрос 24:
Согласно одной книге, будучи выходцем из знатной семьи, Иван Тургенев
познакомился с ИКСОМ на улице. Звучит, наверное, странно, но в "ИКСЕ"
всего 50 слов. Что мы заменили на ИКС?

Ответ:
Русский язык.

Комментарий:
В доме Тургеневых говорили на французском языке. Во втором предложении
имеется в виду стихотворение в прозе "Русский язык" авторства того же
Тургенева.

Источник:
   1. Литературная матрица. Учебник, написанный писателями. Том 1.
http://www.flibusta.net/b/215508/read
   2. http://ru.wikisource.org/wiki/Русский_язык_(Тургенев)

Автор:
Ярослав Косарев (Нижний Новгород)

Вопрос 25:
В одном романе, действие которого происходит в Нидерландах, девушка, с
большим трудом пытающаяся устоять на коньках, сравнивается с НЕЮ.
Назовите ЕЕ.

Ответ:
Мельница.

Зачет:
Ветряная мельница.

Комментарий:
Героиня активно машет руками, словно крыльями мельницы.

Источник:
Нил Стивенсон. Король бродяг. http://www.flibusta.net/b/365885/read

Автор:
Сергей Лобачёв (Нижний Новгород)

Вопрос 26:
Герой романа "Последний конунг" ДЕЛАЕТ ЭТО перед военным походом.
Богатые китайцы, стараясь подчеркнуть свое благородное происхождение,
ДЕЛАЛИ ЭТО крайне редко. Какие два слова мы заменили словами "ДЕЛАТЬ
ЭТО"?

Ответ:
Стричь ногти.

Зачет:
Подстригать ногти; Состригать ногти; Обрезать ногти.

Комментарий:
Герой предчувствует свою скорую гибель и желает всячески отсрочить
постройку Нагльфара. Богатые китайцы ходили с длинными ногтями,
демонстрируя свое пренебрежение к тяжелому труду.

Источник:
   1. Тим Северин. Последний конунг.
http://www.flibusta.net/b/95047/read
   2. http://www.stunningbeauty.ru/index.php/menu-nails-aticles/58-aticle-nails-a4

Автор:
Сергей Лобачёв (Нижний Новгород)

Вопрос 27:
Небольшой английский парк ТвИкенхэм за свое биологическое разнообразие
получил прозвище "ИКС природы". Главный герой известного кинофильма,
увидев ИКС, перестал обращать внимание на окружающую обстановку.
Назовите ИКС.

Ответ:
Конспект.

Комментарий:
В этом небольшом парке наблюдается такое разнообразие флоры, что его
назвали "конспектом природы". Упомянутый известный кинофильм - "Операция
"Ы"".

Источник:
Наталья Юдина. 100 великих заповедников и парков.
http://www.flibusta.net/b/175126/read

Автор:
Ярослав Косарев (Нижний Новгород)

Вопрос 28:
Согласно одной книге, деятельности ЭТОГО ЧЕЛОВЕКА мешали распускавшиеся
слухи, согласно которым у людей могли появиться рога, копыта и хвост.
Назовите ЭТОГО ЧЕЛОВЕКА.

Ответ:
[Эдвард] Дженнер.

Комментарий:
Дженнер прививал людям коровью оспу. Недоброжелатели и скептики
рассказывали о том, что вместе с оспой человек может стать обладателем и
других атрибутов коровы.

Источник:
Елена Грицак. Популярная история медицины.
http://www.flibusta.net/b/113382/read

Автор:
Ярослав Косарев (Нижний Новгород)

Вопрос 29:
Египетского бога Нила по имени Хапи изображали с НЕЙ, что было знаком
плодородия. Может прозвучать странно, но у матадора РивьЕро Эль ПримЕро
была тайна, заключавшаяся в избавлении от НЕЕ. Назовите ЕЕ двумя
словами.

Ответ:
Женская грудь.

Комментарий:
Ривьеро Эль Примеро - это женщина, которая ради карьеры матадора
выдавала себя за мужчину. От груди ей пришлось избавиться, потому что
иначе ее тайна была бы быстро раскрыта.

Источник:
   1. Мэрилин Ялом. История груди. http://www.flibusta.net/b/359437/read
   2. http://www.maximonline.ru/skills/sex/_article/putevoditel-po-zhenskoj-grudi/

Автор:
Ярослав Косарев (Нижний Новгород)

Вопрос 30:
Основатели Яндекса Аркадий ВОлож и Илья СегалОвич, поступив в московские
вузы, написали письмо своему бывшему классному руководителю. Забавно,
что, описывая степень своего недовольства столичной жизнью, они
упомянули ЭТО. Напишите ЭТО, используя пять символов.

Ответ:
10^100.

Зачет:
Гугол; 100^50.

Комментарий:
Провинциальная школьная жизнь Воложу и Сегаловичу нравилась намного
больше, чем московская студенческая. Десять в сотой степени - это число
"googol", от названия которого произошло название компании "Google". Как
известно, "Google" и "Яндекс" - конкуренты.

Источник:
Дмитрий Соколов-Митрич. Яндекс.Книга.
http://www.flibusta.net/b/379792/read

Автор:
Ярослав Косарев (Нижний Новгород)

Вопрос 31:
При описании голландских лугов Всеволод Овчинников упоминает ЕГО. На
одном фешенебельном судне ОН оборудован электроприводом и высокоточными
гироскопами, что позволяет использовать ЕГО даже во время качки.
Назовите ЕГО.

Ответ:
Бильярдный стол.

Комментарий:
Большая часть территории Нидерландов лишена гор и возвышенностей,
поэтому ее зеленые луга напомнили Всеволоду Овчинникову ровную
поверхность бильярдного стола. На круизном судне "Сияние морей"
бильярдные столы оборудованы механизмами, практически полностью
компенсирующими качку.

Источник:
   1. http://www.rg.ru/2011/08/04/yaponcy.html
   2. http://www.geek.com/geek-cetera/gyroscope-used-for-self-leveling-cruise-ship-pool-table-1467721/

Автор:
Сергей Лобачёв (Нижний Новгород)

Вопрос 32:
В сериале "Демоны да Винчи" представитель Ватикана предлагает художнику
предать Флоренцию в обмен на доступ в секретный архив Ватикана. Далее
этот герой сравнивает себя с ИКСОМ. Назовите ИКСА, используя дефис.

Ответ:
Змей-искуситель.

Комментарий:
А ватиканский архив при этом сравнивается с древом познания.

Источник:
Сериал "Демоны да Винчи", s01e02, 47-я минута (в озвучке FOX).

Автор:
Ярослав Косарев (Нижний Новгород)

Вопрос 33:
В самом начале легенды о Поле Баньяне - гигантском дровосеке из
американского фольклора - говорится, что потребовалось целых пять ИХ.
Назовите ИХ одним словом.

Ответ:
Аисты.

Комментарий:
Чтобы доставить такого крепыша родителям, одного аиста явно не хватило
бы.

Источник:
http://www.americanfolklore.net/folklore/2010/07/the_birth_of_paul_bunyan.html

Автор:
Сергей Лобачёв (Нижний Новгород)

Вопрос 34:
Ответьте двумя словами, что король Фридрих II Гогенцоллерн, правивший в
восемнадцатом веке, назвал "артишоком, который предназначен для того,
чтобы его съели лист за листом".

Ответ:
Речь Посполитая.

Комментарий:
В восемнадцатом веке было три раздела Речи Посполитой, в одном из
которых успел поучаствовать Фридрих.

Источник:
Ниал Фергюсон. Цивилизация: чем Запад отличается от остального мира.

Автор:
Ярослав Косарев (Нижний Новгород)

Вопрос 35:
(pic: 20140522.jpg)
   Почтовое управление Китая выпустило серию марок, одну из которых мы
вам раздали. Другая марка этой серии содержит фрагмент известного
произведения. Назовите автора этого произведения.

Ответ:
[Альбрехт] Дюрер.

Комментарий:
Точки на спине черепахи образуют магический квадрат. Другой магический
квадрат изображен на известной гравюре Дюрера "Меланхолия".

Источник:
   1. http://www.futilitycloset.com/2014/10/16/stamps-and-math/
   2. https://www.macaupost.gov.mo/Philately/XVersion/ProductList.aspx?admcode=MAC&emicode=201408&lang=en-us

Автор:
Сергей Лобачёв (Нижний Новгород)

Вопрос 36:
Действие романа "КриптономикОн" происходит во время Второй мировой
войны. Рассказывая об уроженцах островка Йглм [ийглым], расположенного у
берегов Великобритании, герой романа упоминает другую народность.
Назовите эту народность.

Ответ:
Навахо.

Комментарий:
У островитян был свой собственный язык, который был известен только
йглымцам. Этот факт позволил использовать жителей острова для передачи
секретных сообщений. Сходную роль в ВМФ США выполняли индейцы племени
навахо.

Источник:
Нил Стивенсон. Криптономикон.

Автор:
Сергей Лобачёв (Нижний Новгород)


FreeBSD-CVSweb <freebsd-cvsweb@FreeBSD.org>